Download as pdf or txt
Download as pdf or txt
You are on page 1of 52

BRIDGE

Number: 214 October 2020

Julian Pottage’s Double Dummy Problem


FOUR ACES
♠ 3 2
♥ A 3 2
♦ Q
♣ A Q 6 5 4 3 2
♠ K Q J 10 9 ♠ 8 7
♥ K Q J 10 ♥ 9 8 7

N

W E

♦ 10 S
♦ K J 9 8 7 6 5
♣ J 10 9 ♣ K
♠ A 6 5 4
♥ 6 5 4
♦ A 4 3 2
♣ 8 7

Contract 3NT by South


Lead: ♠K

BRIDGE is ceasing publication.


The last issueThewill
answerbe number
will be 216
published on page - month.
4 next December 2020.
See page 5 for more information.
A

Sally Brock Looks At Your Slam Bidding

Sally’s Slam Clinic

Where did we go wrong? Slam of the month


Another regular contributor to these Playing standard Acol, South would This month’s hand was sent in by
pages, Alex Mathers, sent in the open 2♣, but whatever system was Roger Harris who played it with his
following deal which he bid with played it is likely that he would then partner Alan Patel at the Stratford-
his partner playing their version of rebid 2NT showing 23-24 points. It is upon-Avon online bridge club.
Benjaminised Acol: normal to play the same system after
2♣/2♦ – negative – 2NT as over an
opening 2NT, so I was surprised North Dealer South. Game All.
Dealer West. Game All. did not use Stayman. In my view the ♠ A 9 4
♠ J 9 8 correct Acol sequence is: ♥ K 7 6
♥ A J 10 6 ♦ 2
♦ K J 7 2 West North East South ♣ A 9 7 6 4 2
♣ 8 6 Pass Pass Pass 2♣ ♠ Q 10 8 6 3 ♠ J 7
♠ Q 4 3 ♠ 10 7 5 2 Pass 2♦ Pass 2NT ♥ Q 9 N ♥ 10 8 5 4 2
W E
♥ 7 4 3 N ♥ 9 8 5 2 Pass 3♣ Pass 3♦ ♦ Q J 10 9 5 S ♦ K 8 7 3
W E
♦ 8 5 4 S ♦ Q 9 3 Pass 6NT All Pass ♣ 8 ♣ Q 5
♣ Q 10 9 4 ♣ J 5 Once South has shown 23 HCP or so, ♠ K 5 2
♠ A K 6 North knows the values are there for ♥ A J 3
♥ K Q slam. It might be better (from North’s ♦ A 6 4
♦ A 10 6 point of view) to consider 6♦, but 6NT ♣ K J 10 3
♣ A K 7 3 2 should be OK.
Alex asked if declarer should guess
the ♦Q correctly. When he played the The bidding was:
They bid: hand West led a spade and declarer
did well to play the ♠J from dummy, West North East South
West North East South which held the trick. Now declarer is 1♣
Pass Pass Pass 2♦ safe even if he loses a diamond trick. 1♠ 2♠ Pass 2NT
Pass 2♥ Pass 2NT On a less helpful lead, declarer should Pass 6♣ All Pass
Pass 3♦ Pass 4NT start by cashing four rounds of hearts
Pass 5♦ Pass 5NT (pitching a spade and a club). If West North-South were playing Acol. The
Pass 6♦ Pass 6NT throws a diamond (as is likely) I would 2♠ cue-bid showed club support,
All Pass be inclined to play East for the ♦Q. and the 2NT rebid was 15-18. North
could have prolonged the auction but

Mr Bridge 2021 diaries did not want to give away any more
information. 6♣ was a reasonable
punt.
Only £14.95 each Declarer won the diamond lead,
or 10 for £55 & pro rata. ruffed a diamond, drew two rounds of
trumps and ruffed his last diamond.
With a ball-point pen and soft Kidrell He now played ace, king and another
cover in a choice of colours; navy blue, spade. West had to either give a ruff-
and-discard or broach the heart suit.
ruby red, bottle green. They were the only pair to bid the
( 01483 489961 slam.
Well done. n

Page 2 BRIDGE October 2020


BRIDGE
The Mr Bridge Monthly Magazine
Features this month include:
1 October Double Dummy Bridge Problem
by Julian Pottage
ADVERTISERS’
INDEX
2 2021 Luxury Diaries
Ryden Grange, Knaphill, 2 Sally's Slam Clinic 3 Clive Goff ’s Stamps
Surrey GU21 2TH 8 Double Dummy Tea
4 September Double Dummy Bridge Problem Answer Towels
( 01483 489961
by Julian Pottage 11 Duplicate Bridge Rules
Simplified
bridge@mrbridge.co.uk 4 Mr Bridge 13 Little Voice
www.mrbridge.co.uk 5 BRIDGE Ceasing Publication Notice 30 Clive Goff ’s Stamps
shop: mrbridge.co.uk/shop 32 Q Plus 15
6 David Stevenson Answers Your Bridge Questions
33 2021 Luxury Diaries
Publisher/Managing Editor
7 Bidding Quiz by Bernard Magee 37 Advanced Declarer Play
Mr Bridge
with Bernard Magee
9 Declarer Play Quiz by David Huggett
38 Mr Bridge Playing Cards
9 Defence Quiz by Julian Pottage 44 Acol Bidding with
Associate Editor Bernard Magee
10 Acol Two Bids And Strong Bids In Benjamin Acol
Julian Pottage 45 Defence with Bernard
julian@mrbridge.co.uk by Andrew Kambites Magee
14 The Sheriff 's Overbidding by David Bird 50 Declarer Play with
Bernard Magee
Accounts 16 Simple Take-Out Doubles. Part 4 by Sally Brock 51 Advanced Acol Bidding
Jo Jackson with Bernard Magee
18 Catching Up with Sally Brock
52 Exploring the Capitals of
19 Simple Take-Out Doubles. Part 4 Quiz by Sally Brock the Baltic on board
Fred. Olsen's Balmoral
Cartoons & Illustrations 19 Acol Two Bids And Strong Bids In Benjamin Acol
Marguerite Lihou
www.margueritelihou.co.uk
Quiz by Andrew Kambites
20 Julian Pottage explains his Double Dummy Titles REDUCE THE
21 Bridge Glossary – O by Julian Pottage COST OF YOUR
Technical Consultant POSTAGE
Tony Gordon 23 Bridge Glossary – P by Julian Pottage
British postage stamps
32 Bridge Glossary – Q by Julian Pottage for sale at 90% of face-
33 Seven Days by Sally Brock value, all mint and with
Typesetting full gum.
Jessica Galt 35 Julian Pottage Answers Your Bridge Questions
jessica@mrbridge.co.uk Quotations for
37 Simple Take-Out Doubles. Part 4 Quiz Answers commercial quantities
by Sally Brock are available on
Proof Reading Team 39 Defence Quiz Answers by Julian Pottage request.
Julian Pottage Values supplied in
40 Readers' Letters
Mark Rixon 100s, higher values
Mike Orriel 41 Declarer Play Quiz Answers by David Huggett available, as well as 1st
Richard Wheen 43 Acol Two Bids And Strong Bids In Benjamin Acol and 2nd class.
Quiz Answers by Andrew Kambites ( 020 8422 4906
Office 45 Bidding Quiz Answers (1-3) by Bernard Magee or 07719753767
( 01483 489961 46 The Diaries Of Wendy Wensum 8 clive.goff@london
Jessica Galt rugby.com
jessica@mrbridge.co.uk 47 Bidding Quiz Answers (4-6) by Bernard Magee
48 When Is A New Suit At The Three Level Forcing?
by Julian Pottage
Printed in the UK by 49 Bidding Quiz Answers (7-9) by Bernard Magee
The Magazine
Printing Company 50 Letter From Overseas by John Barr
www.magprint.co.uk
51 Bidding Quiz Answers (10-12) by Bernard Magee

BRIDGE October 2020 Page 3


COLD COMFORT REFUNDS
Do read the notice on the
adjacent page. There are
only 100 or so outstanding

FF AND SMOOT refunds but it would be so

RU H tidy if this painful chore


could be quickly completed.
♠ A J 2
♥ 10 9 8 7 6 Readers have kindly
♦ Q contributed a further
♣ 5 4 3 2 £13,427.53 to Cancer
♠ K 10 9 N ♠ 8 7 All events, great or small, Research UK and £8,401.66
♥ Void
W E
S ♥ 5 4 3 2 have a life of their own. to Little Voice Foundation.
♦ A J 9 8 7 6 5 4 3 2 ♦ 10 The national wind-down of Well done them.
♣ Void ♣ K J 10 9 8 7 holidays, cruises and
♠ Q 6 5 4 3 weekends precipitated by MIND SPORTS
the covid-19 pandemic
♥ A K Q J CENTRE
♦ K being no exception. The
♣ A Q 6 continuous cancellations I have dedicated this
are well-documented in the months carrier sheet to a
national press and the synopsis of a long-overdue
Mr Bridge offerings have all project. Give David Parry
Contract 6♥ by South but disappeared. a call if you are minded to
Lead: ♦A help or are in need of more
This has resulted in my information.
unplanned retirement and ( 07710 653 786.
the redundancy of my office
team. Only Jessica is left to 2021 DIARIES
keep BRIDGE going to
12. RUFF AND SMOOTH © Mr Bridge ( 01483 489961. Printed in the UK on 100% cotton.

RUFF AND SMOOTH


press until December. These diaries are £14.95
each. They come in three
TRULY SUREAL cover colours; ruby red,
The Solution to September’s At the end of August we
navy blue and bottle green.

Double Dummy Bridge Problem had an outdoor socially


distanced party which
served as a goodbye
1. Since a spade switch would allow declarer five get-together for all my
redundant office team. Lots
easy spade tricks, West continues diamonds at of tears and love and
trick two rememberances mixed up
together.
2. You discard a spade from dummy, as does They each have a useful
East, while you ruff in hand I was presented with a ballpoint pen in their spine.
lovely sponge cake which The club offer is 10 for £55.
3. Cash three top hearts was consumed with gusto,
the surplus icing and OVERSTOCKS
4. Lead a low spade and finesse the ♠J chocolate being used to
5. Draw the last trump, discarding the low club decorate my youngest Unused stamps (41p & 24p
granddaughter's 1st or 34p & 31p) 2nd class rate.
from hand, and cash the ♠A birthday cake. 65p for 55p per pair, saving
6. Finesse the ♣Q and ruff a spade to leave the 10p on face rate. Also first
class stamps for 67p each.
South hand high ( 01483 489961 for details.
Offered in multiples of 100.
Any variation from the above results in defeat
All good wishes
either because you lose a spade or because East

gets to ruff the second round of spades. n Mr Bridge

Page 4 BRIDGE October 2020


BRIDGE is ceasing publication.
The last issue will be number 216 - December 2020.
I require your action.
Please help me by responding to this notice
as I am keen to account for all monies that are
outstanding, no matter how small. ( 01483 489961.
Thank you to those who have already responded.

Option 1

Donate your remaining balance to Mr Bridge's favourite small charity; Little Voice Foundation

Little Voice works directly with an impoverished


urban community in Addis Ababa in Ethiopia to
support the economic development of women,
to provide primary education to children and
vocational opportunities to youth. We take a holistic,
community-centered approach to ending the cycle
of poverty.

Option 2

Donate your remaining balance to Mr Bridge's favourite large charity; Cancer Research UK

Cancer Research UK funds scientists, doctors and


nurses to help beat cancer sooner. We also provide
cancer information to the public.

Every step we make towards beating cancer relies


on every pound donated.

https://www.cancerresearchuk.org

Option 3

Request a refund of any money owing after the final issue.


Please have your payment card to hand to process the refund.

BRIDGE October 2020 Page 5


David Stevenson Answers Your Bridge Questions

How Should You Deal With


An Exposed Pass Card?

Q
Playing face to face after the lead is faced on the and asked West his strength England and simultaneously
last week (we are previous hand and did not of points for the 2♣ bid. by George Rapee (Samuel
out of lockdown do so thus has committed an West said “2♣ – any Stayman’s partner) in the
where I live) I was East infraction. Thus to treat the strength”. In fact West had USA. At that time it was used
and had just moved for pass card as a call out of turn 1 point; 2 spades, 3 hearts, as either a signoff in clubs
the next round. As we all seems possible. Whether he 5 diamonds, headed by or as a game try or better to
studied our cards I decided has pass values or not is, as the jack, and 3 clubs. find a four card major. There
to bid clubs when it was you suggest, unauthorised What I would like to know: was thus no guarantee of
my turn. I looked up and information to his partner but A) Should West any points whatever. Despite
saw that North had a pass authorised to the opponents. have alerted his 2♣ increasing uses over the
card on the table in front This position is not a bid at the start? years it has not been a bid
of him so opened 1♣. matter of law since the B) Was his explanation that promises any points.
At that moment I realised use of bidding boxes is a “2♣ – any strength” a true For example holding:
that in fact South was the matter of regulation. If there explanation of what turned
dealer. It turned out that was a regulation covering out to be a McKendrick type
North’s pass card had this position of course the of bid, weak in a minor? ♠ x x x x
remained on the table since director must follow it but Apparently, a while back, ♥ x x x x
the previous round but had there is no such regulation. this was accepted in a ♦ x x x x x
been hidden by his score Thus I would not say that Stayman bid but something ♣ Void
card. When he picked this an alternative ruling by the I had never heard of.
up as we sorted our hands director is necessarily wrong We missed game because
the pass became exposed, either. The problem is that it we were not sure of the Most authorities will tell
but nobody noticed. seems unfair to say that the explanation and I wonder you that a response of
The director was called pass has not been made at how you would have ruled 2♣ to 1NT is clear.
and ruled that North had all so East’s 1♣ is a call out if called to our table after It is an unfortunate fact
passed out of turn but that of turn since East-West are the board had been played that Stayman is usually
I had accepted his call by the non-offending side. So I and we found out that taught as a game try or
bidding 1♣, so the auction prefer the director’s ruling. West only had 1 point. better. This is so as not
continued without penalty. We did call the online to confuse beginners.
Everyone at the table ♣♦♥♠ Director who said he was Nevertheless as players’
was happy and thought it correct to say ‘Stayman’, knowledge continues

Q
was a sensible ruling, but I was playing no more, no less, so that most players realise that
was it the correct one? online in a friendly was the end of that. all Stayman promises is
Also, North might have tournament run by Gail Davis, the ability to handle any
had greater values than a bridge holiday company. Merlin BC, Haverfordwest. replies. Therefore, while
his pass suggested (he East opened 1NT. I West was wrong not to write

A
didn’t, in fact) – would (South) had 14 points and Of course West Stayman in the Explain box
this be unauthorised a flat hand, so passed. should have once he had explained his
information to South? West bid 2♣ with no alert announced his 2♣ bid bid as any strength that
Andrew Elliott, (as you probably know you as Stayman. This has been is normal for Stayman.
Onchan, Isle of Man. alert your own bids online). the case for many years. None of this has anything
North bid 2♠, East passed Online he merely writes to do with McKendrick which

A
The ruling looks and I bid 3♠ inviting game. Stayman in the ‘Explain’ box is a different convention
reasonable to me. At this point North and presses the alert button. in a different sequence.
North is required to thought there must be a lot Stayman was invented
pick his bidding cards up of points around the table around 1938 by Jack Marx in Continued on page 8... u

Page 6 BRIDGE October 2020


Bernard Magee’s
Acol Bidding Quiz
This month we are dealing with hands that hold an 8-card suit.
You are West in the auctions below, playing Standard Acol with a weak no-trump
(12-14 points) and four-card majors. It is your turn to call.

1. Dealer East. Love All. 4. Dealer West. N/S Game. 7. Dealer North. N/S Game. 10. Dealer North. Game All.
♠ 2 ♠ 2 ♠ 4 ♠ 4 3 2
N N N N
♥ 7 2 ♥ 7 ♥ 8 2 ♥ 4
W E W E W E W E
♦ 4 3 S ♦ K 9 8 7 6 5 4 3 S ♦ 7 6 S ♦ K 8 7 6 5 4 3 2 S
♣ K J 7 6 5 4 3 2 ♣ J 4 3 ♣ A 10 9 7 6 5 4 3 ♣ 7

West North East South West North East South West North East South West North East South
1♠ Pass 4♦ Dbl Pass 4♠ 2NT Pass 3NT 3♥ 4♥* Dbl
? ? ? ?
* strong two-suited hand

2. Dealer West. Game All. 5. Dealer North. N/S Game. . Dealer East. Love All.
8 11. Dealer West. Game All.
♠ 3 ♠ 8 7 6 ♠ 4 ♠ A K Q J 10 4 3 2
N N N N
♥ A K Q J 9 8 7 2 ♥ 7 ♥ 8 2 ♥ 4 3
W E W E W E W E
♦ 4 2 ♦ 4 ♦ 7 6 ♦ A K Q
S S S S
♣ 8 2 ♣ K 10 8 7 6 5 4 3 ♣ A 8 7 6 5 4 3 2 ♣ Void


West North East South West North East South West North East South West North East South
? 1♠ Dbl 2NT* 1NT Dbl 2♣ Pass 2♦ Pass
? ? ?
* spade raise with 10+ points

3. Dealer East. Game All. 6. Dealer West. Love All. 9. Dealer West. Game All. 12. Dealer East. Game All.
♠ 2 ♠ Q J 8 7 6 5 4 3 ♠ 4 ♠ Void
N N N N
♥ K Q W E
♥ A K W E
♥ 8 2 W E
♥ A 2 W E
♦ 4 3 S ♦ A 3 S ♦ 7 6 S ♦ 7 6 5 S
♣ K 8 7 6 5 4 3 2 ♣ 6 ♣ K 8 7 6 5 4 3 2 ♣ J 8 7 6 5 4 3 2

West North East South West North East South West North East South West North East South
3♠ Pass 1♠ Pass 1NT Pass 3♠ 2♣ Pass
? ? ? 2♦ Pass 3♠ Pass
?

My Answers: My Answers: My Answers: My Answers:

1........................................... 4........................................... 7........................................... 10..........................................

2........................................... 5........................................... 8........................................... 11..........................................

3........................................... 6........................................... 9........................................... 12..........................................

Answers on page 45 Answers on page 47 Answers on page 49 Answers on page 51

BRIDGE October 2020 Page 7


David Stevenson Answers Your Bridge Questions

Double Dummy
Tea Towels t ...Continued from page 6. each team. Consider that a
team whose average is 13 is

Q
Can you advise me likely to do pretty well against
Printed on 100% on a different kind a team whose average is
premium white cotton. of question from nine and will nearly always
the ones you usually get? do far, far better against
31” x 20” (78.74cm x 50.8cm) We’re thinking of running a team whose average is
£7.95 each. an online Teams of Four six, perhaps multiply the
league in Cornwall, but difference by 3, assuming 24
Two or more for £6 each. there will be quite a wide boards, does that sound fair?
range of abilities. So, to
keep everyone in with a Ave 9 v ave 13 = 3x4 =
chance of doing well, some 12 IMPs over 24 boards
sort of a handicap system Ave 6 v ave 13 = 3x7 =
seems appropriate. 21 IMPs over 24 boards
Using current NGS grading Ave 6 v ave 9 = 3x3 =
for each team would 9 IMPs over 24 boards
seem the best solution.
Say, one team had an This might be a little unfair
average of 57%, and on top teams. Perhaps
another 52%, how best could you should call all aces
we reflect the 5% difference 14. Also you have to do it
in ability? Has anyone pro-rata roughly for longer
ever translated percentage or shorter matches.
difference into a number of So suppose you are
IMPs before? Our matches playing 14 board matches,
will probably be 16 boards. perhaps a factor of 2 not 3.
Have you any thoughts,
or is this one for the EBU? Ave 9 v ave 13 = 2x4 =
Chris Bickerdike by email. 8 IMPs over 14 boards
Ave 6 v ave 13 = 2x7 =

A
Since the EBU do 14 IMPs over 14 boards
not run handicap Ave 6 v ave 9 = 2x3 =
tournaments I doubt 6 IMPs over 14 boards
they can help you. Of course
they run tournaments which Having not played in or
are limited to certain levels of run such a competition I
players but that is different. am merely guessing.
I think the simple answer is The nearest I came to
that you have to guess what this was in Manchester
seems fairest. Call the cards where I played in last year’s
2, 3, 4, ... 10, 11 for jack, 12 knockout which was given
for queen, 13 for king, 14 handicaps based on which
for club ace, 15 for diamond division they played in.
More designs available: ace, 16 for heart ace, 17 for
spade ace and sum them for
I would think the above
figures were comparable. n
www.mrbridge.co.uk/shop
( 01483 489961 Email your questions for David to:
Mr Bridge Limited davidstevenson@mrbridge.co.uk
Please include your postal address.

Page 8 BRIDGE October 2020


DEFENCE QUIZ

DECLARER



by Julian Pottage
PLAY

(Answers on page 39)

Y
ou are East in the defensive positions below, playing
QUIZ matchpoints with neither side vulnerable. While you
usually aim to beat the contract, you may also need to
consider the risk of conceding overtricks.
by David Huggett
(Answers on page 41)

Y
ou are South as declarer playing rubber bridge or teams,
which means you are trying to make your contract with
little concern for overtricks. In each case what is your play
strategy?
1. ♠ J 10 8 4 3. ♠ J 10 8 4
♥ K ♥ K
♦ K Q 10 ♦ A K 10 7 4
1. ♠ K 8 6 5 3. ♠ A 9 4 2 ♣ Q J 9 6 4 ♣ K Q J
♥ K J 7 ♥ 6 4 3 ♠ 7 2 ♠ A 7
N N
♦ 7 6 4 2 ♦ Q 10 9 4 ♥ Q 8 7 6 ♥ 8 7 5 4
W E W E
♣ K 6 ♣ A 8 S ♦ A 7 6 2 S ♦ Q 6
♣ 7 3 2 ♣ A 9 8 5 4
N N
W E W E
S S
West North East South West North East South
♠ A Q J 10 7 2 ♠ K Q 10 8 6 1NT1 Pass 1♦ Pass Pass
♥ 6 2 ♥ A 7 5 Pass 2♣ Pass 2♠
1
1♥ Dbl 3♥ 3♠
♦ A J ♦ A J 2 Pass 4♠ All Pass All Pass
♣ J 5 3 ♣ Q 7 1
12-14 and Stayman

Partner leads the ♦9. What is Partner leads the ♣10. What
You open 1♠ fourth in hand You are declarer in 4♠. West
your plan? is your plan?
and find yourself in 4♠. West leads the ♣6. How do you
leads the ♦K. How do you plan the play?
plan the play?

2. ♠ Q 10 2 4. ♠ 6 5 2. ♠ 10 7 4 4. ♠ K 9 5
♥ K 7 4 2 ♥ J 8 7 ♥ K Q ♥ K J 6
♦ Q 3 ♦ Q J 10 5 4 ♦ J 8 6 ♦ Q J 8 7 2
♣ K 9 8 2 ♣ A J 2 ♣ K Q 10 8 4 ♣ K 10
♠ J 3 ♠ J 6
N N N N
W E W E
♥ A J ♥ A Q 10 8 3
W E W E
S S S ♦ A K Q 10 7 2 S ♦ 5 4
♣ 9 6 2 ♣ A 9 7 2
♠ K 5 4 ♠ A K
♥ A 6 ♥ A 10 2
♦ K J 10 9 2 ♦ 9 8 6 West North East South West North East South
♣ A 6 4 ♣ K Q 8 5 3 Pass 1NT1 2♥ 2♠
Pass 1♣ 1♦ 1♠ All Pass
Pass Pass 2♦ 2♠ 1
12-14
You are declarer in 3NT You are declarer in 3NT.
All Pass
after East has overcalled in West leads the ♠7. How do
spades. West leads the ♠7. you plan the play?
How do you plan the play? Partner leads the ♦3. What is Partner leads the ♥9 covered
your plan? by the ♥J. What is your plan?

BRIDGE October 2020 Page 9


About Bidding Judgement by Andrew Kambites

Acol Two Bids And Strong


Bids In Benjamin Acol

S
trong hands have traditionally Now consider a holding like A-J- ♠ = 5.5 tricks: A
-K-J = 2.5 tricks &
been evaluated by their ‘playing 10. Again you intend to take two 6-card suit = 3 tricks
tricks’ rather than point count. finesses. You can expect one to fail ♥ = 2.5 tricks: A-Q-10 = 2 tricks &
and one to succeed (when the missing 4-card suit = 0.5 tricks
Playing Tricks honours are divided between your two ♦ = 1 trick: A = 1 trick
Playing tricks are calculated as follows: opponents). Again you are likely to ♣=0
make two tricks. However A-Q-10 is
Suits with at least three cards: clearly better than A-J-10 because you
3 tricks A-K-Q do have the chance on a lucky day of Hand C
2.5 tricks A-K-J A-Q-J three tricks. In this way Playing Tricks ♠ 10
2 tricks A-K-x A-Q-10 are not a precise tool, though less ♥ 10 9
A-J-10 K-Q-J imprecise than the losing trick count if ♦ A K Q J 7 6 5 4
1.5 tricks A-Q-x K-Q-x you fail to make suitable adjustments. ♣ 9 6
K-J-10 Equally the definition does assume
1 trick A-J-x A-x-x there are entries to your partner’s
K-J-x Q-J-10 hand that allow you to take finesses. Hand C has 8 playing tricks:
0.5 tricks K-x-x Q-J-x A strong hand needing finesses rarely ♠=0
Q-10-9 makes as many tricks as might be ♥=0
expected opposite a really bad hand. ♦ = 8 tricks: A-K-Q = 3 tricks &
In addition, add length tricks as To give some examples: 8-card suit = 5 tricks
follows: ♣=0
0.5 tricks Each 4-card suit
1.5 tricks Each 5-card suit Hand A Acol two bids
3 tricks Each 6-card suit ♠ A Q 6 5 4 3 Acol two bids (2♦, 2♥ and 2♠),
4 tricks Each 7-card suit ♥ A Q J showing near game hands, have been
5 tricks Each 8-card suit etc. ♦ K 8 7 traditionally defined in terms of
♣ 6 playing tricks rather than point count.
Doubletons and singletons If the hand meets the following three
2 tricks A-K conditions it satisfies the requirements
1.5 tricks A-Q Hand A has 7.5 playing tricks: of an Acol two bid.
1 trick A-x K-Q ♠ = 4.5 tricks: A-Q-x = 1.5 tricks & C1) It should have at least 8 playing
Singleton A 6-card suit = 3 tricks tricks.
0.5 tricks K-J K-x ♥ = 2.5 tricks : A-Q-J = 2.5 tricks C2) It should contain sufficient
♦ = 0.5 tricks: K-x-x = 0.5 tricks defensive strength to convince you
This is all very logical. Consider why a ♣=0 that the hand belongs to your side
holding like A-Q-10 is counted as two and that if opponents try to outbid
tricks. Assuming you have sufficient you they will be punished. Thus you
entries to the hand opposite, you can Hand B can afford to bid constructively rather
finesse the ten, and subsequently you ♠ A K J 7 6 5 than feel you need to pre-empt your
can finesse the queen. If both finesses ♥ A Q 10 6 opponents.
are successful you make three tricks. ♦ A 4 2 C3) You have a suit of quality: at
If both finesses fail you make just one ♣ Void least five cards in length. A 5-card
trick, the ace. Half of the time one suit should be very good: A-Q-J-9-x is
finesse will work and the other will minimum. The minimum for a 6-card
fail, giving you two tricks. Hand B has 9 playing tricks: suit should be A-J-x-x-x-x. If partner

Page 10 BRIDGE October 2020


has a doubleton honour (eg K-x) he East might want to pass 2♠ rather than agreement’ in the context of Benji
should feel the suit should usually seemingly dig his side deeper into Acol. Acol two bids should also be
play for at least 5 tricks with at most trouble, but if he plays 2♠ as forcing forcing to suit agreement.
one loser. Note this strict requirement he has to give a negative 2NT response
for suit quality explains why many (0-7 points). When West rebids 3♥ Benjaminised Acol (or
hands are opened 2NT with shapes East realises he has found a huge heart Benji)
that might not be considered ideal (eg fit and he can raise to the excellent Benji is rightly popular with club
5-4-2-2 with a poor 5-card major). If heart game. If you play an Acol two players. The concept is straightforward:
you show a two-suited hand with your opener as forcing to suit agreement (ie 2♥ and 2♠ are played as weak two
rebid that suit may be only four cards the same suit bid twice by the partners) bids (5-9 or 6-10 points and a 6-card
in length. you can afford to open an Acol 2♠ on suit). These bids have a pre-emptive
Hand A would not satisfy C1 and hands with great playing strength value. However I will give a couple of
should be opened 1♠. but limited points, keeping the 2♣ warnings.
Hand B is an Acol 2♠ opener. opening bid for hands with 23+ points. First, you have five strong bids when
Hand C has 8 playing tricks but playing Acol: 2♣, 2♦, 2♥, 2♠ and 2NT.
certainly does not satisfy C2. If your You now have only three strong bids.
opponents bid a grand slam in a major Layout E Many club players enjoy the thrill of
suit could you be confident of beating ♠ A K J 7 6 5 2 ♠ 8 3 playing weak two bids and put a lot
it if your ♦A lead was ruffed at trick ♥ A Q 10 6 3 N ♥ J 2 of work into this part of the system.
W E
1? Hand C is not an Acol 2♦ opening ♦ 6 S ♦ 7 5 3 2 Players tend to neglect the strong
bid. You should open Hand C with a ♣ Void ♣ 10 5 4 3 2 bids. Benji uses 2♦ as the strongest
gambling 3NT, showing a solid 7- or possible bid with a 2♥ artificial
8-card minor and no more than a negative response showing 0-7 points.
queen outside. This has two desirable West East A 2♣ opening shows an Acol two bid
effects: it pre-empts your opponents, 2♠ 2NT in an unspecified suit with 2♦ as the
making it necessary for them to enter 3♥ 3♠ negative response. If opener’s long
the auction at the four level if they 4♠ End suit is a major, a 2♣ opening is game
wish to do so and it makes it easy for forcing unless the first response is 2♦
your partner to judge whether 3NT In Layout E West intends to play in and responder’s rebid is 2NT. I would
is likely to be a making contract for game whatever East has but prefers advise that 2♣ be played as forcing
your side. If he thinks 3NT has little to keep an Acol 2♣ opener for a hand to suit agreement after a negative 2♦
chance, he can take it out by bidding with more points. East replies with a response, just like after an Acol two
4♣; this instructs you to pass if your negative 2NT. West introduces his bid if responder has shown no sign of
solid minor is clubs or convert to 4♦ if second suit, forcing because it is a life.
you have solid diamonds. new suit at the three level. East gives
Opinions differ as to whether an preference to 3♠. This can be passed Auction F Auction G Auction H
Acol two opener should be forcing because now spades have been bid West East West East West East
for one round. The argument that twice by East/West. West now raises 2♣ 2♦ 2♣ 2♦ 2♣ 2♦
8-playing tricks opposite rubbish to the excellent game. 2♠ 2NT 2♠ 3♠ 2♠ 2NT
won’t produce game seems persuasive, Below I explain the concept of 3♠ 3♥ 3♠
and sometimes not even 9 tricks, an auction being ‘forcing till suit u
however the counter argument is
that you need to be able open an Acol
two with a two-suited hand. Look at
Layout D which features Hand B.
DUPLICATE  BRIDGE 
RULES  SIMPLIFIED
Layout D
by David Stevenson


♠ A K J 7 6 5
♥ A Q 10 6 N
W E
♠ 2
♥ J 9 8 7 2
(otherwise known as the Yellow Book) £595
♦ A 4 2 S ♦ 7 5 3
FULLY REVISED IN 2017
♣ Void ♣ 10 5 4 3

West East Available from Mr Bridge Limited


2♠ 2NT
3♥ 4♥
Pass ( 01483 489961 www.mrbridge.co.uk

BRIDGE October 2020 Page 11


t are given the rather misleading you must, if asked, describe accurately
Thus in Auction F East has made an explanation that it is ‘strong’. This of your arrangement. To just say ‘Strong’
initial negative response of 2♦ (0-7 course can put opponents off from is not good enough. Note their advised
points) and a second negative of 2NT bidding even if they have most of the description: ‘Either a strong hand or
(0-3 points). 3♠ is not forcing. high cards between them because each eight playing tricks’.
In Auction G East has made an initial one believes that his partner must have To describe a hand as strong it needs
negative of 2♦ but hasn’t made the nothing, leading to Auctions K and L. to conform to (a) or (b).
second negative of 2NT. Therefore 3♠ It is in your interests to understand While the purpose of my explanation
shows 4-7 points and logic determines how the EBU have chosen to resolve is to try to help you avoid problems
it cannot be passed. West's decision to this because otherwise you could with the director, I would stress that
open 2♣ rather than 1♠ suggests he is find that opponents call the director although you are allowed (with proper
worried that if his partner passes then claiming they have been misled. The disclosure) to open a Benji 2♣ with
game might be missed, so 4-7 points EBU have constantly changed their Hand C it is simply bad bridge. If
opposite should be enough for game. regulations as they proved very hard you hold a hand with lots of playing
In Auction H East has given two to enforce and players are genuinely strength but very limited defence your
negative responses, 2♦ and 2NT. 3♥ is confused. priority should be to do your best to
forcing but, when East gives preference I will give the present EBU criteria buy the contract, while making it hard
to 3♠ over 3♥, that can be passed. and then explain the implications. for opponents to get their act together.
The EBU has had to deal with You should be trying to use up lots of
problems arising out of Acol two bids EBU criteria for strong bidding space, while giving partner an
and Benji. The West hand in Layout J two bids accurate description of your hand. That
is the Hand C we considered earlier. To be considered a ‘Strong’ opening leaves partner best placed to decide on
Look at the two auctions shown below. bid or overcall, the minimum the best action, while opponents have
I would certainly not approve of requirement allowed by agreement is to guess at a high level. Good bridge
the East/West bidding but this sort of either or both of: dictates that you should pre-empt with
auction is seen regularly in real life. (a) any hand of at least 16 HCP. such hands.
(b) any hand of at least 12 HCP with
at least five controls. An ace counts Opening strong,
Layout J as two controls, a king counts as one balanced hands
♠ K Q 5 4 control. The principles behind Benji came from
♥ A K 7 6 Partnerships may agree that an America. Weak two opening bids were
♦ 9 3 2 artificial opening (such as 2♣) may easy to use, effective as pre-empts and
♣ A Q be made with a hand that would not rightly popular. Problems had to be
♠ 10 ♠ 9 8 7 3 historically have been considered solved about how to deal with strong
N
♥ 10 9 W E ♥ 5 3 2 worthy of a forcing opening, such as hands but the solutions found by the
♦ A K Q J 7 6 5 4 S ♦ 8 a balanced or semi-balanced hand early advocates of Benji have proved
♣ 9 6 ♣ 10 8 7 5 4 with fewer than 18 HCP, or a hand enduring. By contrast, other ideas
♠ A J 6 2 with a lot of playing strength but used by the early advocates of Benji
♥ Q J 8 4 limited high cards (such as eight have not proved so effective.
♦ 10 solid spades and little else). This must Bridge players love to tell partner
♣ K J 3 2 be disclosed clearly. For example, the exactly what they hold. However,
opening could be described as “Either there must be a purpose behind this,
a strong hand or eight playing tricks namely to arrive at the best possible
Auction K (Playing Acol) in a major”. This applies even if the contract. It might seem impressive to
West North East South minimum agreed strength is in line know exactly what partner has but not
2♦ All Pass with (b) above. if you end up too high. In the early
There is no restriction on the days of Benji it was noted that there
Auction L (Playing Benji) strength of a natural two-level or were effectively three ways of opening
West North East South higher opening bid but similar 2NT: an actual 2NT opening, a Benji
2♣ Pass 2♦ Pass requirements for full disclosure 2♣ followed by a 2NT rebid after a 2♦
3♦ All Pass apply. negative, and a Benji 2♦ followed by a
2NT rebid after a 2♥ negative. Then
I have already explained that Hand I can understand that such legal there were sequences that involved
C doesn’t fit the requirements of language is not necessarily user opening 2♣ or 2♦ and rebidding with
an Acol two but players count their friendly, but I can easily pick out the a jump to 3NT. So many choices!
eight playing tricks and open 2♦ if important implications, namely that Early Benji enthusiasts introduced
playing Acol or 2♣ if playing Benji. opponents must be properly informed. the idea of showing a balanced 19-20,
The problem is that opponents ask You are permitted to agree to open by opening 2♣ and rebidding 2NT.
the meaning of West’s bidding and hands like Hand C with a two bid but However, look at Layout M.

Page 12 BRIDGE October 2020


three level available, which means Auction R Auction S
Layout M there is less room to investigate the West East West East
♠ 9 4 best fit and no space at all to invite 2♣ 2♦ 2♦ 2♥
♥ J 10 9 8 game. Opening 2NT should be a 3NT Pass 2NT 3♥
♦ A J 7 2 reluctant act, only tolerated when you (transfer)
♣ Q 8 7 are afraid of missing game if you open 3♠ 3NT
♠ K J 7 6 ♠ 5 3 2 at the one level and partner passes. 4♠ Pass
N
♥ A Q 4
W E
♥ 5 3 2 The idea of opening 2♣ and
♦ Q 5 4 S ♦ 10 3 rebidding 2NT with 19-20 is not an If you play that the way to show a
♣ A K 3 ♣ 9 6 5 4 2 essential part of Benji. Yes, original balanced 26 points is Auction R, West
♠ A Q 10 8 enthusiasts played it (or some played has to jump to 3NT to show his point
♥ K 7 6 the 2NT opening as the 19-20 hand), count and the bidding is too high for
♦ K 9 8 6 but with any new idea it makes sense to East to investigate whether a 5-3 or
♣ J 10 keep the strengths and try to improve even a 5-4 spade fit exists. 3NT has no
the weaknesses. Most strong players chance on a diamond lead.
abandoned this flawed idea years ago, It works better to do so via Auction
West opens 2♣, East responds 2♦ and but it persists at club level. Players S: East can show his balanced 26 points
West rebids 2NT, which is passed out. often quote me Hand N as an example with a 2NT rebid, allowing East/West
2NT is a truly awful contract. North of a hand with 19 points where game the space to investigate and uncover
will lead a heart, giving declarer two may be missed if you open just 1♣. the 5-3 spade fit.
heart tricks but where does declarer go This is true, but as my earlier articles Jumping the bidding on strong
from there? in this series demonstrated this hand hands to show point count is not
He needs entries to dummy to take is worth more than 19 points. It has sensible bridge. If you have a very
finesses but dummy has nothing. 5-3-3-2 shape, strong intermediates, strong hand why do you want to pre-
Declarer has four tricks and in practice a good club suit that should provide empt your own side? I suggest you
the defenders might be endplayed extra tricks and no unsupported do best to play a system in which
to give him a fifth trick, but that is queens and jacks. If you are playing sequences like 2♣-2♦-3NT or 2♦-2♥-
hardly a success story. A strong hand a 2NT opening bid as showing 20-22 3NT don’t exist. Of course you can
facing a Yarborough usually has huge points, Hand N is easily worth a 2NT choose to play whatever you want but
problems of communications. Note opening bid. my choice would be the following:
that even giving East the ♣Q instead
of the ♣2 would hugely improve 2NT opening bid = 20-22 points
declarer’s prospects. Two club tricks Hand N Benji 2♣ with a 2NT rebid over a 2♦
would become five and after running ♠ A Q 10 negative = 23-24. Not forcing.
the clubs with the 3-2 club break ♥ J 10 9 Benji 2♦ with a 2NT rebid over a 2♥
declarer would be in the right hand to ♦ A 9 negative = 25 upwards. Game forcing.
take a spade finesse. ♣ A K J 10 5
The issue is not whether you can You might ask how you can sensibly
show a balanced 19-20 points by investigate a slam after 2♦-2♥-2NT
opening 2♣ and rebidding 2NT over Another point is worth making here. if it is unlimited, but balanced 27+
2♦. Of course you can if you want. A Theoretically you can open 2♣ or 2♦ counts are rare and, if partner has
more relevant question is to ask why and rebid 3NT to show very strong nothing, game might be a struggle.
would anybody want to open the West hands but consider Layout P. The priority should be to find the best
hand in Layout M at the two level? game contract. n
West would have been far better off to
open 1♠. East would have passed but Layout P
at least this contract has some play. If ♠ Q 6 2 To support Little Voice, send your
West opens 1♠ and East passes, it is ♥ 8 6 used stamps to:
highly unlikely that East/West have ♦ K Q J 8 3 Malcolm Finebaum
missed a good game and that is surely ♣ Q J 6 Flat 8 Mountford House
the main criteria for opening at the ♠ A K J ♠ 10 9 7 5 4 8 Crescent Road
two level. It is also worth commenting ♥ A K 7 N ♥ 5 2 Enfield EN2 7BL
W E
that any sequence where your first ♦ A 2 S ♦ 7 6 4
Little Voice work directly with an
natural bid is 2NT is a slight weakness ♣ A K 7 3 2 ♣ 9 5 4
impoverished urban community in Addis
of any system. After a 1NT opening ♠ 8 3
Ababa, Ethiopia, to support the economic
bid, you can investigate the correct ♥ Q J 10 9 4 3 development of women, to provide
denomination, sign off, invite game ♦ 10 9 5 primary education to children and
or force to game, all below 3NT. After ♣ 10 8 vocational opportunities to youth.
a 2NT opening bid you only have the www.littlevoicefoundation.org

BRIDGE October 2020 Page 13


Robin Hood's Bridge Adventures by David Bird

The Sheriff’s
Overbidding

T
he Sheriff was not greatly West North East South king, both defenders following. His
overjoyed by the unannounced Guy of The Sgt. Jubert next move was the ♦3 from his hand.
appearance at Nottingham Gisborne Sheriff Heade Wilmere When the ♦7 appeared from Gisborne
Castle of Jubert Wilmere. He had 1♠ in the West seat, he studied it with
made the long journey from London Pass 2♠ Pass 4♠ interest. The 6, 5, 4, 3 and 2 of the suit
on King John’s behalf, to check how All Pass were all on view. The ♦7 was the lowest
much tax had been collected in recent spot card out! It was unclear which
months. Gisborne had a standard opening lead defender held the ♥A, but it could not
‘I hope I can rely on you just this available in the club suit. Mindful of be wrong to duck this diamond trick
once,’ said the Sheriff to Guy of the Sheriff’s instructions, he chose the into the safe hand.
Gisborne. ‘Wilmere has asked to more risky lead of the ♥2. With any ‘Play low,’ said Wilmere.
partner me in tonight’s duplicate. luck, this would give the dreaded tax Sergeant Heade had to overtake
Whatever his standard of play may inspector an undeserved extra trick. with the ♦8 and could not continue
be, it will be unappreciated if you and He might then take a lenient view if hearts effectively from his side of the
your partner give our esteemed visitor the recent Castle tax receipts were table. The portly declarer won the
any poor scores.’ below expectations. club switch and drew the last trump.
‘You can trust me, as always, Dummy went down and Jubert When diamonds proved to be 3-3, he
my Lord,’ Gisborne replied. ‘I’ll be shook his head disapprovingly. discarded a heart on the thirteenth
partnering Sergeant Heade. Not the ‘3NT, that’s where we should be,’ he diamond and claimed the contract.
brightest spark in the fire, but I’ll informed his partner. ‘Also, this man ‘Play the ♦9 on the first round, Sir
explain the situation to him.’ here has found a good lead for them.’ Guy!’ exclaimed the Sergeant. ‘He has
The overweight Wilmere, dressed in The Sheriff glared at Gisborne. to lose a diamond to your hand then,
black, turned out to play a reasonable Had he not listened to the earlier and two more heart tricks put him one
but uninspired game. An hour into instructions? down.’
the session, Gisborne and his partner ‘Play low,’ said Wilmere. ‘What a brilliant duck in diamonds
arrived at the Sheriff’s table. This was Sergeant Heade won with the by Jubert!’ declared the Sheriff. ‘Few
the first board they played: ♥10 and switched to a low club. The would have spotted it.’ He turned
declarer won with the ace and drew towards Gisborne. ‘I told you Jubert
two rounds of trumps with the ace and was one of London’s best players,’ he
Dealer South. E/W Game.
♠ Q 7 6
♥ K 8 5
♦ K 5 4 2
♣ 8 7 2
♠ 10 3 ♠ 8 5 2
♥ Q 9 7 2 N ♥ A J 10
W E
♦ Q 9 7 S ♦ J 10 8
♣ J 10 9 4 ♣ Q 6 5 3
♠ A K J 9 4
♥ 6 4 3
♦ A 6 3
♣ A K

Page 14 BRIDGE October 2020


said. ‘You can’t afford to defend so you want, I didn’t hear. The eight, was hopeless dummy?’
carelessly against an expert like that.’ it?’ ‘With my 4-card support, I was
‘But, my Lord,’ protested Gisborne, ‘Your hearing is as bad as your trying to make it more difficult for the
‘this morning you...’ bidding,’ Wilmere reprimanded. It opponents,’ the Sheriff replied.
‘Be silent!’ cried the Sheriff. He
makes no difference, play the two.’ ‘What a lambs-brain you are,’
entered the score on the sheet and When East showed out, the slam was Wilmere continued. ‘They will find
placed the next board in position. down. Declarer ruffed and rescued a it more difficult to defend when the
trick by crossing to the ace of clubs to dummy is very weak? That makes
lead successfully towards his queen. good sense, do you think?’
Dealer North. Game All. He lost a trick in each black suit. Players at the adjacent tables had
♠ 10 4 3 ‘A valuable but expensive lesson for turned to watch this exchange. It was
♥ 8 3 you,’ he informed the Sheriff. ‘To open beyond their experience to witness
♦ A K 8 6 2 the bidding, you need 13 points or a anyone daring to criticize the Sheriff.
♣ A 6 2 good 12.’ ‘With my good hand, I nearly tried
♠ Q 2 ♠ K J 9 8 6 5 Gisborne shared a brief glance with for a slam,’ declared Wilmere. ‘Play
N
♥ 10 9 6 4
W E
♥ 2 the Sheriff. Had declarer led the 8 or low.’
♦ J 5 4 3 S ♦ Q 10 the 6 on the third round of diamonds, Sergeant Heade paused for thought.
♣ 10 9 4 ♣ K J 5 3 he could have thrown his spade loser The lead marked declarer with the
♠ A 7 and set up two diamond tricks for club king, queen and 10 of clubs. If he won
♥ A K Q J 7 5 discards. Why had the Sheriff said with the ace, declarer would be able to
♦ 9 7 nothing on the matter? unblock the king or queen and cross
♣ Q 8 7 With any other partner, every player to the ♣J subsequently for a successful
in the hall would have heard his finesse against the king of trumps.
reaction. If declarer could be kept out of the
West North East South The last board of the round was soon dummy, he would surely lose a trump
Guy of The Sgt. Jubert on the table: trick as well as the three top cards in
Gisborne Sheriff Heade Wilmere the minors.
1♦ 1♠ 3♥ When the ♣2 appeared from East,
Pass 4♥ Pass 4NT Dealer South. Love All. the Sheriff glared at him. This was
Pass 5♥ Pass 6♥ ♠ 10 8 4 no time to dream up some fancy
All Pass ♥ Q J 8 4 defence. Had the witless Gisborne not
♦ 7 6 3 instructed him of his duty against this
Gisborne led the ♠Q and down went ♣ J 7 4 opponent?
the dummy. Jubert Wilmere eyed ♠ J 7 3 2 ♠ 9 6 5 Wilmere won with the ♣Q and
it disapprovingly. ‘You have a card ♥ 7 5 N ♥ K 3 continued with the ♣K. Sergeant
W E
missing?’ he queried. ♦ Q 8 4 2 S ♦ A K 10 5 Heade knew from the opening lead of
The Sheriff looked down at his ♣ 9 8 5 ♣ A 6 3 2 the ♣9 that declarer still held the ♣10.
dummy in alarm. ‘No,’ he replied ♠ A K Q He ducked once more, retaining his
‘The hand has 3-2-5-3 shape, that’s 13 ♥ A 10 9 6 2 ace.
cards.’ ♦ J 9 Excluded from the dummy now, it
‘I meant that one more high card ♣ K Q 10 was the end of the road for declarer.
is missing,’ Wilmere persisted. ‘You He played the ace of trumps, failing
open with only 11 points, vulnerable? to drop the king, and was soon one
Er, well, not normally,’ stuttered the West North East South down.
Sheriff. ‘But you played the last hand Guy of The Sgt. Jubert ‘I can tolerate such a hopeless
so well, I decided to bid boldly.’ Gisborne Sheriff Heade Wilmere partner no longer,’ Jubert Wilmere
Wilmere won with the spade ace 1♥ declared. He indicated the East chair
and drew trumps in four rounds, Pass 2♥ Pass 4♥ to the Sheriff. ‘You will change places
discarding two spades from dummy. All Pass with this man here for the rest of the
The only apparent chance was to find evening.
the diamond suit breaking 3-3. Both Gisborne led the ♣9 and the Sheriff The round was completed and the
defenders followed to the ace and king laid out his meagre dummy as players nearby could not believe their
of diamonds. attractively as possible. eyes, as the Sheriff left the table with
‘And play the two of diamonds,’ said ‘The Lord preserve me from ever Gisborne in tow.
Wilmere. playing bridge in Nottingham again!’ Wilmere looked across the table at
The Sheriff’s mouth fell open. Had Jubert Wilmere exclaimed. ‘In London his new partner. ‘You defended well on
Wilmere not noticed the high cards we respond only with 7 points or a the last deal,’ he informed him. ‘To play
that had fallen in diamonds? ‘Sorry, good 6. Does it not mark disrespect with you, instead of that bonehead... it
partner,’ he said. ‘Which diamond did for your partner to lay down such a will be a big relief for me.’ n

BRIDGE October 2020 Page 15


About Doubles by Sally Brock

Simple Take-Out Double


Part 4
I
n this article, we will look at take- If partner responds 2NT, then the doubles and you have hearts, bidding
out doubles after openings at the doubler should bid their lowest four- 2NT and then 3♥ is the weakest option,
two level or higher. card suit (some people prefer to play bidding 3♥ directly is invitational and
that the doubler should always bid 3♣ if you have more than four hearts you
a. At the two level without significant extra values – it need to jump to 4♥ if you wish to insist
Generally speaking, doubles of weak can work better on some hands but on game.
two openings are more or less the worse on others). I’m sorry that has been rather a
same as doubles of one-level openings. lot of theory, but you can refer back
The difference lies in responding to (2♥) Dbl (Pass) 2NT to it while looking at the following
such doubles. While a double of a one (Pass) 3♣ four or more clubs, examples.
level opening allows a simple response no significant Suppose your LHO opens 2♥,
with, say, 0-8 HCP, a jump with extras partner doubles and the next hand
roughly 9-11, and a cue-bid (or jump 3♦ four or more passes.
to game) with 12+, that doesn’t work diamonds,
with a double of 2♠, say. You would get no significant
too high and might bypass 3NT when extras Hand A Hand B
that is your best place. 3♥ strong hand, ♠ 8 7 2 ♠ A 10 3
The solution is to play a convention probably with ♥ Q 3 2 ♥ J 6 5
called Lebensohl. Basically, how it four spades ♦ 9 8 4 3 ♦ Q J 10 7
works is that a 2NT response to the 3♠ significant extras ♣ J 5 4 ♣ Q 6 5
double is artificial, showing first and with at least
foremost a weak hand. That frees up a five spades
simple response in a suit to show what 3NT at least 20 HCP Hand A is very weak, so bid 2NT,
would be equivalent to a jump response intending to pass anything partner
after a one-level opening, ie about (2♥) Dbl (Pass) 2NT bids (except a cue-bid, obviously).
9–11 HCP. Because the 2NT response (Pass) 3♣ (Pass) Pass 0–8 HCP, at least Hand B is a different kettle of fish,
is forcing it also gives different ways three clubs and worth a positive bid of 3♦, showing
to make a cue-bid – either directly or 3♦ 0–8 HCP, fewer about 9–11 HCP.
after an initial 2NT response. Here is a than three clubs, This time, again your LHO opens
simple structure: four or more 2♥, partner doubles and the next hand
diamonds passes:
(2♥) Dbl (Pass) 2♠ natural, roughly 3♥ shows four spades
0–8 HCP and a heart
2NT Lebensohl, stopper Hand C Hand D
as above 3♠ invitational with ♠ K Q 10 6 5 ♠ K J 10 4
3♣/3♦ natural, five spades ♥ A 5 4 ♥ 8 7 3
roughly 9–11 (any 3NT natural, but ♦ 10 5 ♦ A Q 10 3
bid now by the probably only one ♣ J 8 7 ♣ K 6
doubler is forcing heart stopper and
to game) very playable in at
3♥ shows four least one minor – With Hand C you want to show an
spades with no suggesting that invitational hand with five spades.
heart stopper partner should not You do this by starting with 2NT and
3♠ forcing with five pass with a then bid 3♠ when partner bids 3♣/3♦.
or more spades singleton heart With Hand D you have only four
3NT to play spades but you don’t hold a heart
4♣/4♦ natural and Note that if the opponents open 2♠ stopper. You show this by making
forcing you have slightly less room. If partner a cue-bid of 3♥. If partner does not

Page 16 BRIDGE October 2020


have a heart stopper himself and does double is more flexible. Partner might hold such a suitable hand, but that is
not have four spades, he will bid 4♣ bid 3NT which would suit you well, or the kind of hand your partner should
and you can then bid 4♦ to offer an he might bid 4♠ which would be OK play you for. He will tend to bid 4♠ on
alternative trump suit. with you. Neither of those outcomes is most hands with four, and otherwise
likely if you overcall 4♦. pass unless he has good distribution.
b. Opponents open at the Now suppose your LHO opens 3♥, He should not remove out of weakness,
three level partner doubles and the next hand so if he bids at all he should have in the
In this case we have even less room and passes. You hold one of the hands region of 10 HCP.
there is inevitably a lot of guesswork. below: Hand J is a typical double of 4♠.
The only advice I can give is that if Partner should pass unless he has
one opponent pre-empts and the other good distribution.
does not raise, then very often 3NT Hand G Hand H Now suppose that your LHO opens
will be the best place for you to play. ♠ J 7 6 2 ♠ Q 3 4♥, partner doubles and the next hand
Obviously if partner doubles and you ♥ K 4 ♥ 10 9 4 3 passes. You hold:
have the other major then you can ♦ A 6 2 ♦ A Q J 10 5
bid that suit, but often you have some ♣ 10 9 6 5 ♣ K 4
scattered values– maybe half a stopper Hand K Hand L
and no good suit. It’s better to take a ♠ A 10 3 2 ♠ A 10 4
chance on 3NT than to bid a poor suit With Hand G, you are not bereft, but ♥ 8 7 2 ♥ 8 7 2
at the four level – at least if you are in partner is playing you to have 7 or 8 ♦ K 4 3 ♦ K 9 8 4 3
3NT, when you make it you will score HCP anyway when he dares to double ♣ Q 5 2 ♣ Q 5
a game bonus. at the three level. Bid a simple 3♠, and
Suppose it is Love All and your RHO expect him to raise you on most hands
opens 3♥. What do you bid? that will make game. With Hand K you should bid 4♠ and
Hand H does not quite have a heart hope for the best. Your partner should
stopper, but any singleton honour have four spades, so your best guess
Hand E Hand F with partner will give you a stopper, or is to play in 4♠. However, with Hand
♠ K J 4 ♠ K 3 2 indeed if that singleton honour is with L you should pass. It could be right
♥ A 3 2 ♥ 5 your RHO. You have good playing to play in 5♦ but partner has made
♦ A Q 10 3 ♦ A Q J 10 4 3 strength in diamonds, so take a chance no promises about diamond support.
♣ K 9 8 ♣ A Q 4 on the heart suit and bid 3NT. You should have either a 6-card suit or
two 5-card suits before you remove a
c. Opponents open at the double without spades. And the same
With Hand E you should bid 3NT. A four level distributional requirement applies if
typical pre-empt shows a long good If an opponent opens four of a partner makes a double of a 4♠ opener.
suit with little outside. So here, you minor, double should be for take-out, So, assuming you have agreed
can duck the heart lead once or twice concentrating on the majors. There to play a double of 4♥ in the
as seems appropriate and then with is not a great deal more to say on the way suggested, what do you bid
any luck your RHO won’t be able to get matter – just hope you guess well. on this hand if RHO opens 4♥?
the lead while you lose the lead to your There are many different views about
LHO in order to set up your tricks. If doubles of four of a major. What I like
your RHO had opened 1♥ you would is for a double of 4♥ to be for take-out, Hand M
have overcalled 1NT, and it may seem and to usually promise four spades. ♠ A 3
rather a lot to bid 3NT all on your own On the other hand, given a double of ♥ K J 10 3
now. And, indeed, it might backfire, 4♠ forces partner to the five level, I like ♦ A 6 5
but that is why people pre-empt – to a double to show a hand like a strong ♣ K 7 6 2
make you guess. The way to look at it no-trump with little wastage in the
is this: You have 17 HCP, which leaves suit opened.
23 HCP for the other three players. On The answer is that you have to pass
average they will have about 8 HCP and take your plus score. Yes, you can
each, so with any luck you will have Hand I Hand J almost certainly beat 4♥ by a couple
about 25 HCP between you and your ♠ K J 10 4 ♠ A 6 5 of tricks, but if you double partner is
partner. It would be wrong to double ♥ 6 ♥ K J 10 3 almost certain to remove to something
on this hand because your partner is ♦ A Q 6 5 ♦ Q 6 you probably will not make. Just pass
unlikely to have a heart stopper and ♣ K J 10 3 ♣ A K 4 3 and take your plus score. If you pass in
then you won’t be able to play in 3NT. tempo, you never know, partner might
With Hand F you could overcall double for take-out.
4♦, but that goes past 3NT if partner Hand I is an ideal double of a 4♥ Next month we will look at doubles
has a good stopper or two in hearts. A opening. Obviously you won't always of 1NT. n

BRIDGE October 2020 Page 17


Catching Up
with Sally Brock

W
e’ve been out and about could not put my hand anywhere near North's 2♦ cue-bid supposedly showed
quite a lot in the last few the glass counter, but in the chocolate three-card heart support. As he
weeks. We went to Dawlish shop or the wine shop no one was couldn't bid 1NT without a diamond
for the weekend to stay with my friend wearing a mask. stopper, perhaps a simple raise to 2♥
Gilly, her boyfriend Vas, daughter Mel Otherwise the online bridge goes would have been about right?
and Mel’s friend Soozy. They usually on. We played the last match in the South understandably thought
let the house out but bookings have Lockdown League, comfortably in North had a better hand and so over
been down due to Covid and so it was the middle of the field – no chance of 4♥ he asked for key cards with 4♠ and
available. It took about four hours to winning or being relegated. We had bid the slam over the one key card
drive there on the Friday, and we just hopes of gaining on this board: reply.
got there in time for an online session West led a diamond, ruffed by
with my Dublin women. Then it was declarer. He then led a heart to the
fish and chips for supper and we Dealer North. E/W Game. king and a heart back, capturing the
played cards (not bridge) into the wee ♠ K 10 3 queen with the ace. After that he led
hours. The following day we went to ♥ K 4 the ♣7 to the king and finessed a club
Exmouth by train and ferry, walked ♦ 8 6 4 2 ruffed by West. A second diamond
a long way and had lunch out before ♣ K 9 6 5 was played, ruffed by declarer and he
going to a favourite pub for dinner. On ♠ J 7 6 2 ♠ 9 5 drew the last trump.
N
Sunday we went out and about, again ♥ 10 8 5 3 W E
♥ Q6 At this point it was a question of how
by train and ferry, before going out ♦ Q 9 7 3 S ♦ A K J 10 5 best to play the spade suit for no losers.
to a fish restaurant for dinner. Such a ♣ 3 ♣ Q 8 4 2 In fact he had an inferential count:
treat to go to a restaurant. We stayed ♠ A Q 8 4 East was known to have started with
on till Monday – Indian takeaway this ♥ A J 9 7 2 two hearts, four clubs, presumably five
time – and again played cards into the ♦ Void diamonds and so only two spades. So
small hours. ♣ A J 10 7 the ♠A followed by a spade to the ten,
A few days afterwards, Briony and I the ♠K and another club finesse would
went to Calais for an overnight visit. have brought home the slam. However,
Pre-Lockdown we did this regularly, What would you bid with that South declarer played for East to have the ♠J
stocking up on wine and cheese and hand after East opens with 1♦? This for his opening bid and the slam was
generally having a nice time together. was the auction at our table: one trick short. We lost 6 IMPs instead
We stay at the Hotel Meurice which, of gaining 13.
for the price, is my favourite hotel West North East South We played in other bridge events:
in the world! All very ornate, with Pass 1♦ 1♥ we are not doing very well in the
excellent service, very good value (€62 All Pass Midsummer Swiss Teams; I have
for a double room), and a convenient started playing on Friday evenings
location in the middle of town. We go I led my singleton club, solving one of usually with Kitty (but I had one game
out to a fish restaurant and discover declarer’s problems, and the contract with Olly) which is enjoyable (but
that in France you have to wear a mask made with four overtricks. In the rather slow); Barry and I both played
to go into a restaurant – they have other room the bidding was: in the YC’s Summer Party Individual
them on sale for €1 if you don’t have – here they arranged each table to
one – and that you need to wear it to West North East South Zoom so it was very sociable – and we
get to the table, and later if you want to Pass 1♦ 1♥ finished OK, but nothing special.
get up – to go to the loo, for example Pass 2♦ Pass 2♠ Chris and I scored well when we bid
– but you don’t have to wear it when Pass 3♦ Pass 4♣ this slam:
you are seated. In the shops it was a bit Pass 4♥ Pass 4♠
variable. In my favourite cheese shop I Pass 5♣ Pass 6♥
was told a mask was 'obligatoire' and I All Pass Continued on page 20... u

Page 18 BRIDGE October 2020


Sally Brock’s
Simple Take-Out Acol Two Bids And

Double Part 4 Strong Bids In


Quiz Benjamin Acol Quiz
(Answers on page 37) by Andrew Kambites
(Answers on page 43)
What do you bid on the following
hands with the auctions given? 1 Consider the following hands 2NT opening bid = 20-22 points
The vulnerability and scoring method (i) How many playing tricks does the Benji 2♣ with a 2NT rebid over a 2♦
should not affect your answers. hand have? negative = 23-24. Not forcing.
(ii) How many points does the hand Benji 2♦ with a 2NT rebid over a 2♥
have? negative = 25+. Game forcing.
(iii) How many controls does the In Auctions 2A to F is the last bid forcing?
Hand 1 Hand 2 hand have? (Ace = 2, King = 1)
♠ K 7 6 2 ♠ A Q 10 7 6 (iv) Under EBU regulations are you Auction 2A Auction 2B Auction 2C
♥ A Q 4 ♥ 7 6 2 allowed to describe the hand as West East West East West East
♦ 7 6 3 2 ♦ A Q 6 5 strong? 2♦ 2♥ 2♣ 2♦ 2♣ 2♦
♣ A 10 ♣ 4 (v) At love all what would you open if 2♠ 2NT 2♠ 3♣ 2♥ 2NT
you are playing Acol with strong 3♠ 3♠ 3♦ 3♥
twos?
West North East South (vi) At love all what would you open if Auction 2D Auction 2E Auction 2F
2♥ Dbl Pass you are playing Benji? West East West East West East
? 2♣ 2♠ 2♣ 2♦ 2♣ 2♥
3♠ 2♥ 2♠ 2NT
Hand 1A Hand 1B 3♠
♠ A Q J 10 8 5 ♠ K Q J 10 6 5
Hand 3 Hand 4 ♥ Void ♥ Void 3 You are playing Benji. What is your
♠ Q 3 ♠ K Q 10 6 5 ♦ A K 6 5 4 ♦ A Q J 5 4 opening bid with these hands. If you
♥ J 10 5 4 ♥ A 5 4 ♣ 7 4 ♣ 7 4 open 2♣ or 2♦ what is your rebid
♦ A Q 3 ♦ 10 5 after a negative response?
♣ 7 6 3 2 ♣ J 8 7
Hand 1C Hand 1D
♠ A K Q 5 4 ♠ K 6 4 3 2 Hand 3A Hand 3B
West North East South ♥ A K 5 4 ♥ A J ♠ Q 10 9 ♠ A Q 4
3♥ Dbl Pass ♦ A K 2 ♦ A K ♥ A K 10 ♥ K Q J
? ♣ 3 ♣ A Q 5 4 ♦ K Q J 6 5 ♦ A Q 5 2
♣ A 10 ♣ A K 3

Hand 1E Hand 1F
Hand 5 Hand 6 ♠ A K Q J 7 6 5 2 ♠ 7 Hand 3C Hand 3D
♠ 7 ♠ 8 ♥ 7 6 ♥ 7 6 ♠ J 8 5 4 3 ♠ Q 6 4
♥ A Q 4 3 ♥ A Q 10 6 5 ♦ 9 4 ♦ A K Q J 7 6 5 2 ♥ K J ♥ A K 4
♦ K 10 5 4 ♦ K 10 6 5 4 ♣ 7 ♣ 9 4 ♦ A K Q 3 ♦ K Q 7 6 5
♣ J 10 3 2 ♣ 7 3 ♣ A K ♣ A J

2 You are playing Benji as I have


West North East South described. Hand 3E Hand 3F
4♠ Dbl Pass Suit bids forcing to suit agreement ♠ A Q 8 ♠ A Q 8
? after a Benji 2♣ opening and no ♥ K Q 8 2 ♥ K Q J 9
values shown by responder. For ♦ A 5 ♦ A K
bidding strong, balanced hands you ♣ A K J 10 ♣ A K J 10
have the following agreement:

BRIDGE October 2020 Page 19


Catching Up
with Sally Brock
Julian Pottage explains his Continued
Double Dummy titles. t ...Continued from page 18.

GRIDLOCK Dealer East. E/W Game.


The difficulty is of moving through the play without being stuck ♠ K J 7
in the wrong place. ♥ 6 4 2
♦ A K 10
PROTECTION ♣ Q J 8 7
The theme is protecting from an overruff as South ruffs spades ♠ 10 4 ♠ 9 8 3 2
in hand. ♥ A K 9 7 5 N ♥ Q 10 8 3
W E
♦ 8 7 4 3 S ♦ 9 5 2
THE EXCHEQUER ♣ 5 2 ♣ 9 4
Sorry my history was slightly wrong. John Morton from whom ♠ A Q 6 5
the play Morton's Fork takes his name was Lord Chancellor ♥ J
rather than Chancellor of the Exchequer. ♦ Q J 6
♣ A K 10 6 3
RED BALLOONS
The two red nines are key cards in the play and there was a
song 99 Red Balloons. West North East South
Pass 1♣
THE EDWARDIAN Pass 1♦ Pass 1♠
The key play is variation on a Scissor's Coup and there was a Pass 2♥ Pass 3♦
film Edward Scissorhands. I also have a son Edward, who is a Pass 4♣ Pass 4♥
quiet lad and rarely in the limelight. Pass 4NT Pass 5♣
Pass 6♣ All Pass
ELIZABETH AND MARY
The key play involves queens played as successive cards on the After my partner opened 1♣, I was
key trick and the name refers to Queen Elizabeth of England not sure what our methods were. Did
and Queen Mary of Scotland. we play inverted minor-suit raises or
not? So I temporised with 1♦, Chris
THE GENERATOR bid 1♠ and I needed to bid the fourth-
The play is about generating extra tricks to add to those readily suit in order to make a forcing raise in
available. clubs. When she bid 3♦ I thought she
would probably have the distribution
GROUNDWORK she had. I agreed clubs and over her
Before you begin the crossruff you need to prepare the ground 4♥ cue-bid, tried Roman Key Card
by making your club winners. Blackwood. I would have to admit that
there was little point in this. If she had
PROHIBITION had only two key cards and bid 5♥, we
In the play you try to prohibit East from gaining the lead and would have been too high! It would
prohibit West from discarding a diamond. have been more prudent to bid 4♠,
allowing Chris to bid 4NT, which she
FOUR ACES could have done more safely.
The declarer has side all four aces, each one of which you On a domestic front, things are
need to play at the right time. getting interesting. First, Briony is
going to Kenya for a couple of months
RUFF AND SMOOTH shortly; she has a placement in a
This is a combination of 'ruff and discard' and 'rough and hospital for abused women and
smooth' as well as relating to the fact the in some ways the children (she is doing an Open
cards lie well (black suit finesses) and in another way they lie University degree in psychology). Then,
badly (trump break). the following day, Toby is moving out –
to share a flat with a friend in Stratford
(East London). Things are going to be
very quiet for me! n

Page 20 BRIDGE October 2020


A to Z of Bridge

A to Z of Bridge
compiled by Julian Pottage

O the ♥2 asks for a club.

ODD TRICK
Each trick won by declarer in excess
world compete – now known as the
Mind Sports games.

ONE-BID
of the book. ‘One odd’ is one trick in A bid at the one-level.
excess (ie declarer’s seventh trick).
OBLIGATORY FINESSE ONE CLUB SYSTEM
The play of a small card on the second OFFENCE A system of bidding that employs an
round of a suit in the hope that a 1. Any breach of the Law. artificial bid of 1♣ as its strongest
particular opponent will have to play 2. Attacking mode in bidding or play. opening. Usually the bid denotes a
the master card. For example: minimum of about 16 or 17 points. In
OFFSIDE the UK, strong club systems were quite
If you finesse and it loses, you can say popular in the early 1980s but are now
K 4 3 2 the missing card is ‘offside’. relatively rare. Precision is the most
N
common strong club system.
A 8
W E J 10 9
S
A Q 10 ONE OVER ONE RESPONSE
Q 7 6 5 N
A sequence such as 1♣-1♠, where
K 7 5 W E
S
J842 responder bids at the one-level. A one-
over-one response has a very wide
To avoid two losers in the suit, you 9 6 3 range and (by an unpassed hand) is
start with a small card towards dummy. forcing for one round. Responder may
After winning with the king, you play have as few as 5 or 6 points or, with an
small from both hands – there is no In this double finesse position, the unsuitable hand type for a jump shift,
point in playing up to the queen since king is onside (under the queen) while great strength.
East’s failure to capture the king marks the jack is offside (over the ten).
West with the ace. If you reverse the
East-West hands, it would be necessary OGUST Hand 1 Hand 2
to play initially from the North hand Another name for Blue Club responses ♠ K Q 8 5 2 ♠ K Q 8 5 2
towards South’s Q-7-6-5; again, you to a 2NT enquiry over a weak opening. ♥ 9 6 3 ♥ A K J 4
would need to duck the second round. Opener rebids as follows: ♦ 8 5 ♦ A 4
♣ 7 5 2 ♣ K 3
ODD-EVEN DISCARDS 3♣ minimum, poor suit
A system of discards in which the 3♦ minimum, good suit
face value (odd/even) of the discard 3♥ maximum, poor suit A one-over-one response of 1♠ is
may signal attitude or suit preference. 3♠ maximum, good suit correct with either of the above hands
Odd cards encourage the suit 3NT A-K-Q-x-x-x in suit opened if partner opens one of either minor.
discarded while even cards indicate
suit preference between the other two OLYMPIAD ONE-SUITER
suits. For instance, when discarding A bridge event held every four years, An unbalanced hand with one long
on a spade lead, the ♥3 asks for a in the same years as the Olympics, suit and no other biddable suit. Pre-
heart, the ♥8 asks for a diamond and in which countries from all over the emptive openings and overcalls u

BRIDGE October 2020 Page 21


t show a one-suiter with a single bid. usually a good lead. A suit partner has
Bidding a suit twice, particularly if the K 9 5 bid is usually a good lead too. Against
rebid is with a jump, tends to show a N a no-trump contract, you usually lead
one-suiter. W E
S
A 10 6 2 from a long suit, aiming to set up long
cards as well as high-card winners.
ONLINE BRIDGE When, as often happens, you do
A method of playing bridge using a Holding the ace over dummy’s king not have an ideal holding in any suit,
computer and an internet connection. and the ten over the nine, East is you will need to weigh up, from the
You can play from home or from unlikely to want to open up this suit. auction and your hand, whether to
wherever you have a laptop and a While it might be safe to do so if South prefer a passive lead or an active lead.
suitable connection. You play against holds Q-x or Q-J-x, often you will find If you can see that suits are breaking
other players sitting at their computers. the suit frozen and opening it up will badly for declarer and finesses are
Online bridge is popular for players cost a trick. If South has J-x, x-x, J-x-x, likely to fail, you are more likely to go
who want to play at unsocial hours x-x-x, Q-x-x, Q-x-x-x or Q-J-x-x, it is passive. If the auction sounds confident
or who, for whatever reason, cannot better not to lead the suit. and your hand holds few surprises for
attend a bridge club. declarer, you are more likely to attack.
OPENING BID A trump can be a good lead if you have
ONSIDE The first call of the auction other than a good holding in declarer’s second
If you take a finesse and it succeeds, pass. The generally accepted wisdom suit. Leads from unsupported honours,
you can say the missing card is ‘onside’. is that you need a hand a little better especially aces, tend to be risky.
than average (eg 12+ HCP) to make a
OPEN one-level opening. This way, if partner OPENING LEADER
1. To make the first bid in the auction. has his fair share of the remaining The player who makes the opening
2. A teams or pairs competition points, your side can make a part- lead, the one to the left of declarer.
where no restriction applies to the score, or if partner also has opening
contestants (sex, age, master point bid strength, you can make game. OPTIONAL DOUBLE
ranking etc). A double suggesting all-round
OPENING POINTS strength and inviting partner to choose
OPEN HAND A method of hand valuation by which between bidding on and defending
Dummy. you add your high card points to the for penalties. At low levels, optional
length of your two longest suits. doubles are relatively rare; normally
OPEN PAIRS doubles are either definitely penalty or
A competition open to any pair, definitely takeout. Optional doubles
irrespective of nationality, age, sex Hand 1 Hand 2 tend to occur when the opponents
or masterpoint ranking. In British ♠ A Q 8 7 5 ♠ A Q J 7 5 2 have pre-empted to the game level.
congresses, where entry to the main ♥ K J 7 3 ♥ A K 8 3 2
event is restricted, it is customary for ♦ 7 4 ♦ 5 West North East South
an Open Pairs event to take place at ♣ 6 3 ♣ 4 4♠
the same time. Dbl

OPEN ROOM The first hand has nineteen opening West North East South
Room where spectators can watch the points (10 in high cards, 5 for the spade 1♥ Dbl 4♥
players. length and 4 for the heart length). The Dbl
At international or other important second hand has twenty-five (14 HCP,
events it is usual to have at least one 6 spades and 5 hearts). On each of these auctions, West’s
room for this purpose. double is optional.
OPENING LEAD
OPEN UP The lead to the first trick, before the ORANGE BOOK
To ‘open up’ a suit is to play the first dummy appears. The opening lead is Colloquial term for The Handbook
cards in that suit. As a defender, you often the single most important play of Directives and Conventions
are more likely to open up new suits on a deal. Before making the opening Authorised by The Laws and Ethics
when you are playing an active defence lead, you should consider the bidding Committee of The English Bridge
than when you are trying to defend and your own hand. At duplicate, Union.
passively. having selected your lead, you should
Some suits are safer to open up place it face down on the table. OUT-OF-THE-BLUE CUE BID
than are others. The riskiest suits to The best opening leads are those This is a cue bid made before the
open up are those where you have an that are both safe (unlikely to concede partners have bid and raised a suit. It
honour ranking just above an honour a trick) and attacking (likely to set up implicitly agrees the suit partner has
held on your right. tricks). A sequence such as K-Q-J-x is just bid as trumps and shows a control

Page 22 BRIDGE October 2020


in the suit of the cue bid. North’s 2♣ is an overcall and, in OVERTAKE
bidding 2♣, North is overcalling. To play a higher card from one hand
West North East South An overcall can serve many when already winning a trick. You
1♥ Pass purposes: often use this technique when a hand
2♣ Pass 3♥ Pass 1. To disrupt the opposing bidding; is devoid of entries and the suit is
3♠ 2. To suggest a lead; blocked. For example:
3. To find a good contract in which to
A spade contract is unlikely to be best play;
– East has shown six good hearts and, 4. To pave the way for a sacrifice, ♦ A Q J 6 5 4
by not rebidding 2♠ despite having the which may be cheaper than letting N
strength to reverse, has denied four the opponents play in their contract. W E
S
spades. Therefore, West’s 3♠ agrees Shape and suit quality are more
hearts and shows a spade control. important for an overcall than they ♦ K
Depending upon the partnership are for an opening bid. Here, North
style, the control may be first round could hold a 9-count with ♣K-Q-10-x-
(ace or void) or second round (king or x-x and an ace on the side, but should If dummy has no side entry, you
singleton). not hold a 2-3-3-5 13-count with overtake the king with the ace. This
West might hold: ♣A-J-x-x-x. gives you access to three tricks in the
suit and may allow you to run the
OVERCALLER entire suit if it divides evenly.
♠ A J 4 The player who makes an overcall.
N
♥ K 6 OVERTRICK
W E
♦ 9 6 4 S OVERRUFF A trick in excess of the number
♣ A Q 10 8 4 To ruff a trick that someone has already for which you have contracted.
ruffed, with a higher trump. Although Overtricks do not count towards game
usually it is a good idea to overruff and, in rubber bridge, you score them
OUT ON A LIMB when you get the chance, if you would above the line. For example, if you
Phrase used to describe a dangerous have to play a very high trump to do so bid 4♠ and make 13 tricks, you have
action such as bidding no-trumps and you have a useful discard to make, made three overtricks, the difference
with no stopper in an opponent’s suit. it may be better not to do so. Also, if between the 13 you have made and the
an opponent threatens to overruff you, 10 you contracted to make.
OVER again it might be better to discard. At matchpoints, making overtricks
Term describing a player’s hand (or can be a particularly useful way to
cards therein) with respect to his right- improve your score.
hand opponent, eg ‘Sitting over’. South ♠ Q 9 5 2
is over East while honours in West’s ♥ 9 6 3 OVERTRUMP
hand will be over South’s honours. ♦ A K 5 Synonym for overruff.
♣ 9 7 2
OVERBID ♠ J 10 3

P
♠ 8
A call made on insufficient values. ♥ 10 5 N ♥ A K Q 8 7 2
W E
For example, if partner opens 2NT ♦ Q 8 7 4 S ♦ J 9 2
showing 20-22 points and you raise to ♣ J 8 5 3 ♣ 10 6 4
6NT with an average 10-point hand, ♠ A K 7 6 4
this is an overbid. ♥ J 4
♦ 10 6 3
OVERBIDDER ♣ A K Q
A person who overbids. PACK
The deck of 52 playing cards.
OVERBOARD South plays in 4♠ after East has bid
To be at too high a level. hearts. The defenders start with three PAIR
rounds of hearts. If South ruffs the A partnership of two bridge players.
OVERCALL third round of hearts low, West can
1. The first bid by a member of the side play the ten (or jack) of spades to PAIRS EVENT
that did not open the bidding. overruff. South does better to discard An event in which players compete
2. To make a bid after an opponent has his diamond loser. Then, if East as pairs, normally with matchpoint
opened the bidding. persists with a fourth heart and West scoring.
ruffs with an honour, dummy can play
West North East South the queen to overruff. PALOOKA
1♦ 2♣ A poor bridge player. u

BRIDGE October 2020 Page 23


t the ♦J, which you ruff. You want to duplicate, a 50 bonus applies for any
PAR delay playing on hearts until you have partscore bid and made.
The result on a board if both sides had eliminated the other suits so that
bid and played to the optimum result. if a heart finesse loses to West then PASS
he can only return a heart or give a Call by which a player indicates that he
PAR CONTEST ruff and discard. The snag is that if does not wish, or is not allowed by the
A contest, usually using preset hands you draw two rounds of trumps and Laws, to enter the bidding at his turn
of great technical difficulty, where ruff two clubs, dummy will be out of to bid. Auctions normally end when
players compare their results with the trumps and it will be safe for West to three consecutive players pass. Using
par rather than each other. exit in diamonds. Accordingly, you bidding boxes, the pass card is green.
On each deal usually only one side draw only one round of trumps. Then
has the chance to earn a par with the you cash the top clubs, crossruff the PASS OR CORRECT
other side not involved. The par setters minors and play a heart to the nine A bid made after partner has shown
may specify the bidding or the play to (or cover East’s card). Since East has one or more unspecified suits by
the first trick or two. S uch contests are the outstanding trump, the partial means of a conventional bid that is to
rare in the UK. elimination succeeds. play if and only if the bid suit coincides
with one of partner’s possible suits.
PARTIAL PARTIAL STOPPER For example:
A partscore contract, ie one less than A holding that, if partner has a similar
game such as 2♦. holding, will prevent the run of the West North East South
opposing suit. Typical partial stopper 3NT Pass
PARTIAL DESIGNATION holdings are J-x-x and Q-x. It is the 4♣
An incomplete request for dummy to same as a half stopper.
play a card. If declarer names only the Facing the Gambling 3NT opening,
suit, dummy must play the lowest card PARTNER which usually shows a solid minor and
in the suit. If declarer names only the One of the two members of a little outside, West decides he has too
rank, dummy must play the card from partnership. few stoppers to pass 3NT. He bids 4♣
the same suit as last led. expecting East to pass with a long suit
For example, if dummy holds PARTNERSHIP RUBBER BRIDGE of clubs and otherwise to correct to 4♦.
A-9-4-2 of spades and declarer says Rubber bridge where players retain the
‘spade’, dummy plays the ♠2. If on the same partner throughout the session PASS-OUT
next trick, declarer says ‘nine’, dummy of play. 1. A deal is a ‘pass-out’ if all four
plays the ♠9. players pass at their first turn to
PARTNERSHIP call. At rubber bridge, the deal
PARTIAL ELIMINATION UNDERSTANDING passes to the next player, at Chicago
A play by which a declarer only An agreement between members of a the same player must redeal, and at
partially eliminates the suits that a partnership regarding c onventions in duplicate bridge the hands go back
defender may safely lead. Whether bidding and play, allowing for efficient in the board as if played and the
the defender will have to lead to communication. Such agreements, score is zero.
declarer’s advantage depends on the whether explicit or implicit, must be 2. To make the third consecutive pass
distribution. fully and freely available to opponents. after someone has bid.
Here is an example:
PARTSCORE PASS OUT OF TURN
A trick score of less than 100 points. A A ‘Pass’ by a player when it is not his
♠ K Q 8 2 ‘partial’ is another name. turn to call. The Laws apply.
♥ 7 6 5 3 2
♦ K 7 PARTSCORE, BIDDING TO THE PASS-OUT SEAT
♣ J 6 Bidding affected by the presence of a A player is in the pass-out seat if the
♠ 6 ♠ 4 3 partscore. For example, playing rubber auction would end if he passed.
N
♥ K J 4 W E ♥ 10 8 bridge with a score of 40 below the
♦ J 10 9 6 5 4 S ♦ A Q 8 3 2 line, a 2♥ response to a 1♠ opening PASSED HAND
♣ 9 5 4 ♣ Q 10 7 2 would not be forcing. A hand that has already passed (and
♠ A J 10 9 7 5 therefore, if its holder subsequently
♥ A Q 9 PARTSCORE BONUS makes a bid, that bid is limited in
♦ Void If a rubber ends without two games value by his previous pass).
♣ A K 8 3 scored, there is a 100 bonus for a For example:
partscore in an incomplete game. At
Chicago, there is a bonus of 100 for West North East South
As South, you arrive in 6♠. West leads a partscore on the fourth deal. At Pass Pass 1♥ 1♠

Page 24 BRIDGE October 2020


Here both West and North are passed refusing to open up one of the red PENALTY DOUBLE
hands. No matter what they bid suits, returns a low spade, thereby A double made in the belief that the
subsequently, their partners will not continuing the passive defence. opposing contract will not make.
place them with great strength. Another name for this is a ‘business
PASSIVE LEAD double’.
PASSIVE DEFENCE A lead made more in an attempt For example:
A style of defence that attempts to to avoid conceding a trick than to
avoid establishing tricks for declarer as establish one. A lead from a long West North East South
opposed to actively trying to establish weak suit is usually a passive lead. The 1♥ 1NT
defensive tricks. stronger your hand, the more attractive Dbl 2♠ Dbl
This style is most likely to be right a passive lead. This is because partner
when dummy is flat, lacking a source is less likely to have fitting cards in the West’s double of 1NT is a penalty
of tricks. To defend passively you suits in which you have strength. If double, as is East’s double of North’s
would usually play on suits where the opponents stagger into their final retreat to 2♠.
your holding is worthless or where the contract and you sense that the cards Whether a double is a penalty
previous play of the suit has already are lying poorly for them, this also double is partly a matter of general
determined how many tricks each side indicates a passive lead. bridge knowledge and partly a matter
can make in the suit. of partnership agreement.
PATTERN
This is a shape of the hand as expressed PENALTY PASS
♠ K 8 2 in suit lengths. For example, a hand To pass a takeout or optional double
♥ K Q 9 3 with five spades, a singleton heart, thus effectively converting it to a
♦ A 10 7 5 four diamonds and three clubs has a penalty double. For example:
♣ K 9 5-1-4-3 pattern.
♠ A 10 4 ♠ J 5 West North East South
♥ 10 2 N ♥ J 7 6 5 PENALTY 3♥
W E
♦ Q 6 4 S ♦ J 8 2 1. This is a term for the score that Pass Pass Dbl Pass
♣ Q 10 7 5 3 ♣ A 8 6 4 arises if you fail to make a contract, Pass
♠ Q 9 7 6 3 especially a doubled contract.
♥ A 8 4 2. The sanction imposed by the Laws, East’s double is for takeout, expecting
♦ K 9 3 or by the Tournament Director, for West to bid something. West’s pass is
♣ J 2 an irregularity or an infraction. a penalty pass, indicating a wish to
defend 3♥ doubled.
PENALTY CARD You will need a long strong trump
South plays in 4♠. West leads the five A card that a defender wrongly plays holding to make a penalty pass,
of clubs and the ace wins. Returning or exposes may become a penalty especially if you are under the declarer,
a club to dummy’s now bare king card. In duplicate, some penalty as partner is likely to be short in the
is passive and gives nothing away. cards are designated ‘minor’ penalty suit.
Opening up either red suit would be cards, others ‘major’ penalty cards.
highly dangerous. In rubber bridge, there is no such PERCENTAGE PLAY
If declarer next plays a spade to the distinction. The Laws apply. The line most likely to succeed based
queen, West takes the ace and, again on mathematical probabilities.

BRIDGE
♠ 8 2
♥ 9 8 3
♦ A K 7 5 4
♣ 9 5 2

is ceasing publication. ♠ J 10 9 4
♥ Q 7 4 2 N
W E
♠ A K 5
♥ K 6 5

The last issue will be number


♦ Q 9 S ♦ J 10 8 2
♣ 10 7 4 ♣ 8 6 3
♠ Q 7 6 3

216 - December 2020.



♥ A J 10
♦ 6 3

See page 5 for more information.


♣ A K Q J


As South, you play in 3NT. u

BRIDGE October 2020 Page 25


t West leads the jack of spades and you with only one pair (a half table). In
win the third round with the queen. consequence, on every round one pair ♠ 9 5 3
You have eight top tricks and chances will be due to play the ‘phantom pair’ N
in the red suits for more. If diamonds and will instead spend that round ♠ Q
W E
S
♠ 10 6 4 2
are 3-3, you can set up the suit by sitting out.
ducking a round. If East has one or ♠ A K J 8 7
both heart honours, you can make an PHANTOM SACRIFICE
extra heart trick by using the ♦A-K as A sacrifice bid when the contract
entries for finessing. With no apparent against which one is sacrificing would South might ‘pick up’ West’s queen
way of combining the chances, you fail. Taking out a phantom sacrifice by cashing the ace and then ‘pick up’
need to choose between the two. is often a costly move, turning a East’s ten by finessing, thereby ‘picking
The chance of a 3-3 diamond break plus score into a potentially sizeable up’ the whole suit without loss.
is about 36% while the chance that minus. Before making a sacrifice, it
East holds one or both heart honours is therefore wise to check that you are PICK UP SLIP
is about 75%. The percentage play is confident the opponents can make Used in place of a traveller and collected
the one with the higher probability of their contract. by a caddy during a competition.
success, in this case finessing twice in
hearts. PHONEY CLUB PIN
Note that in practice you would A rudimentary system based on a The lead of an honour to neutralise a
cash some clubs before committing strong no-trump and five-card suit card ranked just below it.
yourself, though as the difference in openings in diamonds, hearts and
percentages is so high, it is unlikely spades, with a (possibly) phoney club
that you would have cause to change being the first move on other hands. A 10 9 8 7
your mind. ‘Fishing Club’, ‘Short Club’ and N
‘Utility Club’ are similar systems. K 6 5 J
W E
S
PERMUTATION
Term describing all possible PHONEY DIAMOND Q 4 3 2
arrangements of a set of objects and, An opening of 1♦ used in much
in bridge, the possible disposition of the same way as a phoney 1♣. You
the cards. will meet this most commonly in The lead of the queen towards the ace
conjunction with a strong club system brings in five tricks when, as here, the
PERSONAL SCORE CARD and five-card majors. If opener cannot jack is singleton and on the right and
The card on which a player may record start with 1♣ because that would show therefore ‘pinned’ by the queen.
all the details of his bridge session, 16+ or 17+ and cannot start with 1♥ or
including the outline of his bidding 1♠ because that would show a five-card PIP
system and conventions, as well as suit, 1♦ will be a common opening. A A design on the front of the playing
contracts played and results achieved ‘Loose Diamond’ is another name for card showing the card’s rank by the
on each board of the session. the method. number of pips, and the suit by the
While keeping a personal score card shape of the pip. Different countries
is not compulsory, it is helpful if at least PICK A SLAM use different images for their pips:
one member of the partnership does Some pairs use a bid of 5NT hearts, cloves, acorns, leaves etc.
so because this will help in resolving (not preceded by 4NT) when the
any scoring queries. partnership has bid more than one PITCH
suit to indicate that the values for a To discard.
PETER slam are present but doubt remains
The play by a defender of a high card about strain. PIVOT TEAMS
on the first round of a suit followed by In general it is a good idea to A teams contest whereby members
a lower card on the second round to establish the trump suit before of each team change partnerships so
signal to partner eg the seven followed embarking on a slam hunt, so the that by the end of the contest every
by the three. Usually you use the peter convention usually occurs only in member of each team has to have
to show that you like the suit or that highly competitive auctions where the played a proportion of the boards with
you have an even number of cards opposing bidding has prevented the every other member of the team as a
in it. Other names for the peter are a customary exploration. partner.
‘high-low’ signal and, especially in
North America, an ‘echo’. PICK UP PLACING THE CARDS
To play a suit with minimal loss, or to The diagnosis of the position of key
PHANTOM PAIR capture a particular card (eg ‘To play cards from clues arising from the
If the number of pairs at a duplicate a small spade to hand picking up the bidding or play.
event is odd, there will be a table queen’). For example:

Page 26 BRIDGE October 2020


The correct order of play is in In 2, this is a side suit in a trump
♠ K 10 8 rotation, clockwise, starting with the contract. If declarer finesses dummy’s
♥ 8 4 3 2 player who leads to the trick. West queen and cashes the ace, West should
♦ A 7 3 plays after South, North after West drop the king.
♣ Q 10 2 and so on. Declarer may then fear an overruff
♠ Q 7 4 ♠ 5 on the third round of the suit. So long
♥ K 7 N ♥ A Q 9 6 5 PLAYED CARD as West retains the king, declarer can
W E
♦ Q 10 6 4 S ♦ 9 8 2 A defender plays a card by placing it safely ruff low.
♣ A J 7 4 ♣ 9 6 5 3 face up on the table in front of him.
♠ A J 9 6 3 2 If a defender places a card in such a PLAYING CARDS
♥ J 10 position that the other defender can The set of fifty-two cards used for
♦ K J 5 see the front of the card, it counts as playing the game of bridge.
♣ K 8 played. The pack consists of four suits,
Declarer plays a card in a similar clubs, diamonds, hearts and spades
manner and any card placed face up, (ascending rank order) each containing
As South, you play in a spade contract on or near the surface of the table, by thirteen cards: ace, king, queen, jack,
after West has opened 1NT (12-14) and declarer counts as played. 10, 9, 8, 7, 6, 5, 4, 3, 2 (descending rank
East has shown a heart suit. Declarer plays dummy’s cards by order). The card rank is indicated by a
West leads the king of hearts and either naming them (the correct pictorial image and/or the number of
continues with a low one. East wins procedure in duplicate bridge) or pips, the suit is indicated by the shape
with the ace and continues with the physically handling them (in rubber and colour of the pips.
nine. From the bidding you place West bridge).
with a heart shortage (rather than the PLAYING TO THE SCORE
king-queen of hearts). PLAYER To allow one’s decisions regarding
Counting the missing points then One of the four people who participate bidding or play to take account of the
enables you to place the rest of the high in a game of bridge. score.
cards. You place West with the ♠Q, the Typically, it means to underbid
♦Q and the ♣A-J. This tells you to ruff PLAYING A KNOWN CARD at rubber bridge with a partscore.
with the ace, finesse West for the ♠Q If a player has the choice between For example, if partner opens 1NT
and then finesse in clubs rather than cards, it is sometimes better to play the showing 12-14 points and you have
diamonds. one that his opponent knows he holds. 13 points, you would raise to 2NT
For example: rather than the usual 3NT if you have
PLAIN SUIT a partscore of 30 or more.
In a trump contract, any of the three
non-trump suits. 1. A J 4 PLAYING TRICK

N
A card that you can reasonably
PLAN OF PLAY Q 10 8
W E 532 expect to win a trick given a normal
S
The mental process that declarers distribution of the cards and ignoring
should follow in determining their best K 9 7 6 the possibility of losing tricks to ruffs:
line of play to make their contracts. A-K-Q-J-x-x is six playing tricks and
As declarer, you would usually make K-Q-J-x-x-x-x-x is seven.
a plan once you see dummy. You may
need to revise your plan as the play 2. A Q 7 5 3 PLUS VALUES
progresses and you find out more These are small adjustments made in
N
about the layout of the opposing cards. K J 9 4
W E 10 8 2 valuing your cards, typically when
S
using the honour tricks method of
PLAY 6 hand valuation.
1. The play follows the auction.
During the play, players contribute POCKET
their cards to tricks, one card to In 1, South, as declarer, plays a small Part of a duplicate wallet or board
each, until they run out of cards. card and successfully finesses the used to hold the cards.
2. Used to describe an action in the jack. When the ace is played West
play, for example, ‘The key play is to should drop the queen (the card he is PODI
ruff a diamond’. known to hold) presenting South with A method of coping with intervention
a choice of plays, finessing East for the after partner’s Blackwood bid. Pass
PLAY OUT OF TURN ten or playing for the even break. shows zero (O) aces. Double shows one
The play of a card by a player when it is If West follows with the ten on the (I) ace and the lowest bid two aces etc.
another player’s turn to play. The Laws second round, declarer will inevitably This method is relatively rare.
apply. play for the break. u

BRIDGE October 2020 Page 27


t position at the table. would fail because East discards after
POINT-A-BOARD 2. Position can be a relative term, such North and would discard the same
A scoring method of determining as a player’s position in relation to suit as North.
a winner in a teams-of-four match. the dealer (first hand is the dealer,
You score each board individually, second hand is on the dealer’s left POSITIVE RESPONSE
getting one point for a win and half a etc.). Position can also be in relation A constructive response guaranteeing
point for a tie. to another player: to sit over/behind some conventionally agreed minimum
In Britain, it is more common to (to be the left-hand opponent) or strength. The term usually applies after
award two points for a win and one for under/in front of (to be the right- partner has made a forcing opening
a tie. hand opponent). bid (when the mere fact of keeping
Tactics are similar to matchpoints 3. The position of a card eg if North the bidding open would not promise
because what counts is whether you holds the A-Q of spades and West strength).
beat the result at the other table rather holds the ♠K, this card is in a For example, 2♣-Pass-2♠ would be a
than the margin of the win. favourable position for North- positive response. Requirements vary
South. between partnerships. Some allow a
POINT COUNT positive response on a four-card suit;
A method of hand valuation by points, POSITIONAL FACTOR others require a five-card suit or even a
the most popular of which is the The value of a particular holding may five-card suit headed by two of the top
Milton Work count: you count four change as the auction develops, thus three honours.
for an ace, three for a king, two for a indicating the likely disposition of the
queen, one for a jack. other relevant cards. POST MORTEM
Some pairs, depending upon the For instance, a holding of K-x in The discussion of a bridge deal after
auction, also count points for length a suit is more valuable if the lead is the conclusion of play. Discussing
or shortages. coming up to it or partner has bid the a deal can be useful in helping
suit; it is less valuable if your left-hand you to learn from any mistakes or
POINTED SUITS opponent has bid the suit. Thus, the misunderstandings.
Diamonds and spades, so called positional factor affects its value. The best time to discuss a deal is
because of the shape of their suit at the end of the session, when hand
symbols. Similarly, clubs and hearts POSITIONAL SQUEEZE records can be useful for the purpose.
are the ‘rounded suits’. A squeeze that is effective against one You should only conduct a post
opponent but not the other, ie it will mortem at the table if you are sure that
POKER BRIDGE not operate if you interchange the you are not holding up or upsetting
A style of bidding and play that has opponents’ hands. the other players.
an unduly large element of gambling A common reason why a squeeze
about it. works against one opponent only is POSTING THE SCORE
if both threats are in the same hand, To place the overall results chart of
PORI which means they will need to lie over an event where it is accessible to the
After an opponent doubles a 4NT ace rather than under the stoppers. competitors.
enquiry, Pass shows zero (O) aces, and For example:
Redouble shows one (I). This method POWERHOUSE
is common for users of PODI but A hand of tremendous trick-taking
otherwise rare. ♠ A J ability.
♥ K
PORTLAND CLUB, THE ♦ — PRECISION
First body to codify the Laws of Bridge ♣ — A bidding system that originated
(1895). At rubber bridge, it is still an ♠ K Q ♠ 9 in Taiwan in the 1960s and was the
important authority in the world of ♥ A N ♥ Q brainchild of C. C. Wei. It achieved
W E
international bridge and holds the ♦ — S
♦ — world attention when the team from
copyright to the Laws in many parts ♣ — ♣ 2 Nationalist China finished second
of the world. ♠ 6 in both the 1969 and 1970 World
♥ 6 Championships. The system is based
PORTLAND RULES ♦ — on an artificial 1♣ (16+ points)
Rules the Portland Club has laid down ♣ A opening, five-card majors and a 13-15
that include banning the use of any 1NT. A 2♣ opening is natural while
conventional bids in the club’s rooms. a 2♦ opening shows a three-suited-
Both threats are in the North hand, hand short in diamonds; 2♥ and 2♠
POSITION so when South leads the ♣A West is openings are weak twos. The range for
1. The cardinal compass point (North- squeezed in the majors. If you swap the 1♥, 1♠, 2♣ and 2♦ openings is 11-
South-East-West) of a player is his the East-West cards, the squeeze 15, ie less than the 16 required to open

Page 28 BRIDGE October 2020


1♣. For raising partner’s 1♥ opening to 4♥ (because opener cannot open a four-
When opener has a strong hand, you might hold: card major) and when the partnership
the benefit of the system is that the is playing a mini or strong no-trump.
partnership can establish a game force
at a low level (any positive response ♠ 7 5 PREPARED HANDS
creates a game force), making it easier ♥ K Q 8 4 3 In a par contest, competitors play
to find the best game or slam. When ♦ J 9 6 4 2 specially selected hands designed
opener has a weaker hand, the benefit ♣ 7 to test particular skills. Before Mr
is that responder knows opener’s Bridge took over its running, the
strength better than in a natural Charity Challenge simultaneous pairs
system. The disadvantages are that PREFERENCE event featured prepared hands. The
a 1♦ opening occurs commonly and After partner has bid two suits, putting organisers invited experts to submit a
says little about opener’s hand; in him back to his first suit or leaving him hand or two.
addition, vigorous opposing bidding in his second suit is giving preference.
can disrupt sequences that start 1♣. For example in the sequence: PREPARED MINOR SUITS
1♥- p a s s -1N T- p a s s -2♦ - p a s s -2 ♥, An opening bid of 1♣ or 1♦ made on
PRE-DEALING responder has given preference for a balanced hand with possibly only
The dealing of the hands in advance hearts over diamonds. Since opener’s a three-card suit to prepare for a no-
of the competition. This is common if first suit will invariably be at least as trump rebid.
a computer does the dealing or if the long as his second suit, responder
competition is a simultaneous pairs. would also show ‘preference’ for PREPAREDNESS, THE
opener’s first suit if his two holdings PRINCIPLE OF
PRE-EMPTIVE BID OR were of equal length. Responder would The principle whereby one selects
SHUTOUT BID ‘prefer’ the second suit by passing or an opening bid so that one always
A weak high-level bid based upon the raising. has a sound rebid over any possible
playing strength contained in a long With 3 cards in opener's second response. The principle can also apply
suit, with few outside values. The bid suit and 2 cards in opener's first suit, to responding hands if responder
is purely obstructive in nature and responder may go back to opener's first intends to bid more than once.
can make opposing bidding extremely suit, an action known as giving 'false
difficult. preference'. This is partly because a PRESSURE BID
Pre-emptive bids can be opening 5-2 fit may play better than a 4-3 fit A bid made at a high level due to the
bids or overcalls: and partly because doing so keeps the presence of interference bidding.
bidding open in case opener is fairly For example after a 1♠ opening
West North East South strong. and a 3♣ overcall, responder may
3♦ have to bid 3♠ with 7-8 points and
PREPARED CLUB some spade support, less than that
West North East South An opening bid of 1♣ made on a required for an uncontested 3♠
1♣ balanced hand with possibly only a bid. Such a bid is a ‘pressure bid’.
3♦ three-card (or even two-card) club suit If holding full values for a
West North East South to prepare for a no-trump rebid. 3♠ raise, responder would find
1♣ 1♥ 1♠ Using a prepared club is quite another bid to take the pressure of
4♥ common in five-card major systems opener, such as jumping to 4♠. u

BRIDGE
In each case, West’s bid is pre-emptive
in nature. For the first and second
auctions, West might hold:



♠ 8 6 2
♥ 5 N
W E
is ceasing publication.
The last issue will be number
♦ A Q J 10 5 3 2 S
♣ 8 3

PRE-EMPTIVE RAISE
216 - December 2020.
A raise based on distributional rather
than high-card values in an attempt to See page 5 for more information.
pre-empt the opponents, rather than
necessarily reach a makeable contract.

BRIDGE October 2020 Page 29


t The usual rule is to allow the makes his living from the game of tables in play. The winners on a round
opposing bidding to pressure you into bridge, though not necessarily as a move up a table while the losers stay
bidding one level higher than you player. put (or vice versa).
would have bid without their bidding.
PRINCIPLE OF FAST ARRIVAL PROBABILITIES OF
The idea that in a game-forcing DISTRIBUTION
♠ K 8 5 4 situation, the more quickly you reach The mathematically expected
♥ Q 4 N game the less interest you have in distribution of the opposing cards.
W E
♦ J 6 S alternative contracts.
♣ A Q 9 8 5 Opponents Division Probability
West North East South hold (to nearest
1NT Pass 2♦* Pass percent)
West North East South 2♥ Pass 3♣ Pass 8 cards 5-3 47%
1♣ Pass 1♠ 3♥/4♥ 4♥ 4-4 33%
? *transfer to hearts 6-2 17%
7-1 3%
You would have bid only 2♠ if South It is common here to play that West’s 7 cards 4-3 62%
had passed, so you should bid 3♠ over 4♥ shows a dead minimum opening. 5-2 31%
3♥ but not 4♠ over 4♥. With a heart fit and any sort of 6-1 7%
willingness to co-operate if East 6 cards 4-2 48%
PRIMARY SUPPORT has slam interest, West would bid 3-1 36%
Four-card support for a bid that 3♥ to set the suit and leave room for 5-1 15%
promises four or five cards in the suit, investigation. 6-0 1%
such as a natural opening bid of one of 5 cards 3-2 68%
a suit. Three cards suffice if partner has PRINCIPLE OF RESTRICTED 4-1 28%
shown six or more cards in the suit. If CHOICE 5-0 4%
partner has made a prepared opening This is the idea that if an opponent 4 cards 3-1 50%
in a minor, you need five cards. makes a particular play, especially 2-2 40%
In most systems, showing primary dropping a high card, this is more 4-0 10%
support for a major fixes the suit as likely to be a forced (ie restricted) play 3 cards 2-1 78%
trumps. rather than a chosen play. 3-0 22%
2 cards 1-1 52%
PRIME VALUES 2-0 48%
Aces, kings and other cards that stand ♠ Q 4 2
a high chance of being useful in the N
PROGRESSIVE SQUEEZE
play. W E A triple squeeze in which you force an
S
opponent to release control of one suit
♠ A K 9 3 and then use the newly created winner
♠ K Q J 10 6 in that suit to squeezed him again in
♥ A K 6 the remaining two suits. For example:
♦ J 6 4 Suppose you cash the ace and cross
♣ Q 4 to the queen, picking up the jack (or
ten) from West on the second round. If ♠ K
the jack (or ten) is a forced play from a ♥ A J
In this hand, the ♠K-Q-J-10 and the doubleton honour, you want to finesse ♦ 3
♥A-K are prime values. The ♦J and the the nine on the third round. Finessing ♣ 5
♣Q, which may or may not contribute is normally correct because with ♠ A
to the hand’s trick-taking potential, J-10-x West might equally have played ♥ K Q N
W E
are not. the other honour on the second round. ♦ K Q S
♣ —
PROFESSIONAL PROGRESSIVE BRIDGE ♠ 5
A bridge professional is a person who A form of social bridge with several ♥ 5
♦ A J
REDUCE THE COST OF YOUR POSTAGE ♣ A
British postage stamps for sale at 90% of face-value, all mint and with full
gum. Quotations for commercial quantities are available on request.Values
supplied in 100s, higher values available, as well as 1st and 2nd class. South plays the ♣A forcing West
( 020 8422 4906 or 07719753767 8 clive.goff@londonrugby.com to relinquish control in one of the
other three suits. South then cashes

Page 30 BRIDGE October 2020


whichever suit West discards to the king South might choose a bid PUNCH
squeeze him again in the other two that makes him rather than North To force a player to shorten his trumps
suits. This type of squeeze can be declarer. During the play, South might by ruffing.
automatic but is more commonly arrange to lose the lead to West rather
positional, as here (East, if holding than East. PUNT
West’s cards, could escape the second To bid directly to game or slam when
squeeze by discarding a diamond). PROTECTION more constructive and descriptive
This means taking action when your bids are available.
PROMOTION side has not been in the bidding and, The term normally refers to hands
The play of one card to promote if you passed, the auction would end. on which you bid a game or a slam
another to winning status. The term In common context, the term is on a speculative rather than a sound
usually applies to covering honours or synonymous with re-opening. basis.
to a promotion in the trump suit that
occurs through ruffing. PROTEST PERIOD PUPPET STAYMAN
The period in competitions, usually for A version of Stayman for five-card and
thirty minutes after the officials have four-card suits following an opening
♠ Q 9 posted the score, within which a player bid of 1NT or 2NT. After 2NT-Pass-
or players may request the correction 3♣-Pass-3♦ (denying a five-card
N
♠ J 10 8 5
W E ♠ K 7 4 3 2 of any anomaly in the scoring or may major), responder bids the major in
S
register an appeal against a ruling by which he does not hold four cards.
♠ A 6 the tournament director. The main purpose of bidding the
major he does not hold is that the
PSEUDO SQUEEZE strong opening hand will become
West leads the jack, which the queen A play that leads a defender to believe declarer if a fit is present.
covers. East covers in turn, causing the that a squeeze position exists when in For example:
promotion of West’s ten to winning fact none does.
rank once South’s ace has gone. Most defenders are inexpert at
defending when they have to make a ♠ K Q 8 2 ♠ J 10 7 4
PROPRIETIES series of discards, so it is often a good ♥ K 8 5 3 N ♥ 4 2
W E
The rules of proper conduct, ethics idea as declarer to run a long suit. ♦ A Q J S ♦ K 8
and etiquette. ♣ A 6 ♣ Q 10 5 4 2
PSYCHIC BID
PROTECT A bid that deliberately and
1. If you have small cards to guard an substantially misstates the strength West East
honour in a suit, you may describe and / or distribution of a hand. So long 2NT 3♣1
the honour as ‘protected’. as the bid is as much of a surprise to 3♦2 3♥3
2. To bid after two successive passes so your partner as the opponents, psychic 3♠ 4♠
that your partner may have another bids are a legitimate part of the game. 1
do you have a five-card major?
opportunity to bid. The word For example: 2
no
protect comes about because your 3
showing four spades
partner may have held adequate
values for a bid but had the wrong ♠ 8 5 4 A heart lead would put 4♠ from the
kind of hand (such as length in the ♥ K 7 3 2 N East seat in jeopardy.
W E
opposing suit) to act on his own. ♦ 8 4 S
With West as declarer, there is the
3. If you prevent an opponent from ♣ Q 8 5 2 chance of playing three rounds of
leading through a vulnerable high diamonds to discard a heart from the
card, you protect that high card. East hand.
For example: West North East South
3♥ Pass PUSH
4NT 1. To make a non-constructive raise
8 6 4 in a competitive situation – usually
Knowing that North-South certainly with the idea to encourage the
N
A 9 7 3
W E Q J 10 2 have a game on and possibly a slam, opponents to bid higher, to a level
S
West feigns a show of strength by where you can defeat them.
K 5 asking for aces. 2. A board in a teams match with no
swing (slang).
PUMP
South’s king is safe from a lead by Colloquialism for forcing declarer to
West, but not by East. To protect ruff. u

BRIDGE October 2020 Page 31


Q PLUS
t QUANTITATIVE 5NT

Q
The use of 5NT, usually as a direct
raise of no-trumps, to request partner
to bid 6NT or 7NT depending on


whether he is minimum or maximum
Still
Only 15 in the context of the previous bidding.
If neither partner has a long suit, you
will need to able to envisage 35-38
£99
including
QUANTITATIVE 4NT
The use of 4NT, usually as a direct
points between the two hands for a
quantitative 4NT.
p&p
raise of no-trumps, to request partner
to pass or bid 6NT depending on QUEEN OVER JACK
The very best Acol- whether he is minimum or maximum An assumption made in rubber bridge
playing software in the context of the previous bidding. that the queen lies over the jack of
available. If neither partner has a long suit, you the same suit more often than simple
will need to able to envisage 31-34 probabilities suggest. It comes from
Features include:
points between the two hands for a the theory that on the previous deal
• a friendly interface quantitative 4NT. the queen may have covered the jack
• a hint button - always at and that, after the trick was gathered,
hand & a help button - the two cards remained together even
explains the features for ♠ A 8 5 2 after the shuffle.
bidding and card play advice ♥ K Q 10
♦ A K 5 QUICK TRICK
• Easy Windows installation
♣ K J 3 A quick trick is the same as or similar
• Rubber bridge, duplicate and to a defensive trick. In one suit, an A-K
teams scoring is two quick tricks; ace or K-Q is one;
• Feed in your own deals If partner shows a balanced 12-14 (for A-Q is one and a half; a king is a half.
example by opening a weak 1NT), you Lower honours do not count towards
• Create your own system
make a quantitative raise to 4NT. quick tricks.
• Pre-programmed systems
include Acol, Standard QUITTED TRICK
American and many more Hand 1 Hand 2 In duplicate bridge, you call a trick
• Instant results for teams ♠ K 10 4 ♠ K Q 4 quitted once all four players have
♥ A 8 5 ♥ A 8 5 turned their cards over. In rubber
• 5,000 hands to play at teams ♦ Q J 9 2 ♦ Q J 9 2 bridge, you call a trick quitted once
of four
♣ Q 7 4 ♣ Q 7 4 the four cards making up the trick
• 4,500 hands to play at are gathered by the winning side.
matchpointed pairs. In duplicate, nobody can inspect a
With the first hand, partner is quitted trick although a player may
For Windows XP, Vista, 7, 8 or 10. minimum and passes 4NT. The second look at (but not expose) his own card
hand has 14 points (maximum) and until someone leads to the next trick.
NEW - Q Plus 15 warrants an advance to 6NT, which is In rubber bridge, a player may inspect
includes Android a good contract. the current trick until his side has
compatibility (5-9) played to the next trick. n

BRIDGE
Download only - no
requirement for a CD-Drive

TRADE IN
Q Plus 15 just £50 if you
have any old bridge software. is ceasing publication.
The last issue will be number
Call for details.
Mr Bridge Limited Ryden Grange,
Knaphill, Surrey GU21 2TH
( 01483 489961
216 - December 2020.
www.mrbridge.co.uk/mrbridge-shop
See page 5 for more information.

Page 32 BRIDGE October 2020


Seven Days
by Sally Brock

Karen and Axel play a defence to a methods), and West removing to 4♥. A
SATURDAY Multi that has gone out of favour in mere +620 would not have scored well
Up early for my weekly session with the UK. 2♥ shows a take-out of hearts. for East-West.
Fiona against Hanna and Siyu – the I am happy to double, asking partner After the game we go in the car
hands are particularly unexciting this to pass with hearts. When he obliges, to Aldeburgh and walk along the
week. Then I pick up Barry before Axel is rather stuck. He bids his 5-card beach, stopping to buy Dover sole for
driving off to Suffolk to stay with suit and I am happy to double that. I dinner at one of the fish shacks there.
his sister Carole. We have a late soup kick off to a good opening lead: the Certainly cheaper than London prices.
lunch and then chat about this and ♠A. Barry ruffs the second spade, We go home for a snacky lunch and
that (she lives alone so apologises for plays a diamond to my jack and I then out again to Sizewell where the
being particularly chatty when she has give him another spade ruff. He plays nuclear reactor is. That may be a blot
company) until a healthy supper of another diamond. I take two tricks on the landscape (though there is a
smoked fish and salad. there, and play another spade and he strange beauty about the bright white
overruffs the dummy. I still have the dome), but the sea is beautiful – huge
SUNDAY ♥A and another couple of trump tricks
to come. Plus 1700. This is worth only
horizon with the sea going on forever.
I sit doing nothing while Carole and
Our regular weekly session with Karen a 6 IMP gain; at both the tables we are Barry cook the Dover sole – which is
and Axel is today. Somehow there is comparing with, East opened a weak completely delicious.
always a big penalty lurking around 2♥, South doubled, West redoubled
the corner when we play them. Here is
this week’s effort:
and North-South were in the same
position. The standard English defence MONDAY
to a Multi would have worked much In the morning I have my usual online
better here, because South would session with Debbie, then we go to
Dealer North. Game All. pass 2♦, expecting to be able to make Richard and Jenni’s house for lunch.
♠ 10 8 6 a take-out double of 2♥ later. On the They are friends who live in London
♥ J 10 9 6 5 actual deal that would have led to West but also have a country house in
♦ 8 5 4 bidding 2NT, East 3NT (showing two Aldeburgh, where they have decided
♣ 8 7 of the top three heart honours in their to be for the lockdown. u
♠ A J 7 4 3 ♠ Q
N
♥ A KQ8432

Mr Bridge 2021 diaries


W E

♦ A Q J 7 S ♦ 9 6 3
♣ K 9 4 ♣ J 10 6
♠ K 9 5 2 Only £14.95 each

or 10 for £55 & pro rata.
♥ 7
♦ K 10 2
♣ A Q 5 3 2 With a ball-point pen and soft Kidrell
cover in a choice of colours; navy blue,
West North East South ruby red, bottle green.
Pass 2♦ 2♥
Dbl Pass Pass 3♣ ( 01483 489961
Dbl All Pass

BRIDGE October 2020 Page 33


t It was great to see them and the and Martin arrive 7ish, and we have improve. In the middle of all of this,
extension they have had built for their a really nice evening: the food works Briony lands in Addis Ababa and texts
house. We have an excellent lunch well and conversation flows freely me from there. Maybe I am distracted
(just managing to eat most of it outside (I’m really impressed about how well by the movements of my children and
before the rain starts) and then go for informed Toby is about US politics). Gilly has not been well. We both feel
a walk in the area beyond their house. so depressed afterwards that we are
We probably outstay our welcome
(though you would never know it) and THURSDAY considering not playing on Sunday.
Maybe we wouldn’t have done if it
it is early evening before we leave. We Today is quiet. I help Briony with wasn’t for this high spot near the end:
only have a little room for soup for various packing decisions. We both
supper and entertain ourselves with help Toby decide what he should
Casualty and University Challenge. take. What is rubbish and what is fit Dealer West. Game All.
for a charity shop? Later on I have a ♠ 10 6

TUESDAY practice with Fiona against Heather


and Maggie – always good fun.


♥ K Q J 6
♦ 9 6 4 3
It’s roughly a three-hour drive to get ♣ K J 9
home and I drop Barry off before
getting back to my place just in time FRIDAY ♠ A 8 5
♥ 2 N
♠ 9 3 2
♥ 10 7 5
W E
to go and pick Briony up from her I 'Zoom' with my old schoolfriends ♦ A Q J 8 5 S ♦ K 10 7 2
cricket training. Then she’s off to for half an hour or so in the morning ♣ 7 4 3 2 ♣ 8 6 5
sailing while I am online with Andy. and then wander up the High Street ♠ K Q J 7 4
In the evening there is a Midsummer – Briony needs to get her Kenyan ♥ A 9 8 4 3
Swiss match which we manage to win shillings and I need a bit of food. We ♦ Void
by 12 IMPs. I don’t join in for the post potter about – she is still deciding what ♣ A Q 10
mortem because Briony is just back to take. There are a lot of uncertainties
from sailing and I feel as if I haven’t about flying in the Covid-19 world.
really seen her for a while (and she is She’s been told she is to be allowed very West North East South
going away on Friday). little hand luggage but as much hold Pass Pass Pass 1♠
luggage as she wants. Should she put Pass 2♦ Pass 2♥

WEDNESDAY her computer in the hold? What about


her camera? Etc. Then we help load the
Pass
Pass
3♥ Pass 4♣
5♥ Pass 6♥
We are entertaining this evening and car up with Toby’s stuff for tomorrow. All Pass
I have forgotten to buy a few things. At six o’clock Briony and I set off for
Briony needs to go and have a Corona Heathrow and when I come back I Gilly has good judgement in the slam
virus test in order to be able to travel play in the YC duplicate with Kitty. zone, and this deal is no exception.
on Friday, so she drops me off in We do some good stuff and some bad She had made a non-forcing 3♥ bid, so
Holland Park and I buy a few things but do at least finish positive. when I still make a slam try of 4♣ she
– most importantly cheese from the knows she has magic cards. As I hadn’t
Jeroboam deli there. (My old favourite
in South Kensington has closed down SATURDAY shown delayed diamond support, I
am likely to be short there. Her jump
so I was looking for a replacement.) Today I am playing in the ‘Eastbourne’ to 5♥ shows good trumps which is
I haven’t had time to do my usual Pairs with Gilly. It is also the day Toby exactly what I want to hear. I bid the
exercise routine, so after she leaves (she is moving out. I had promised to slam happily and lose just the ♠A. This
is helping with a holiday cricket camp help him but have passed that job on scores us 93.4% as only a handful of
for schoolchildren) I get all that done to Barry because of my bridge date. other pairs bid it.
in readiness for an 11.15 online session They leave at 9.30 or so – to Stratford, Later I cook venison steaks for Barry
with Gilly, but she rings to cancel as first to get the keys to the flat, then and me – it’s ages since he’s been here
she is not feeling good. I spend the to move Toby’s stuff in, then to go to as I usually go to his place when the
next hour or so deleting old emails (I Joe’s flat to pick up his stuff, etc, etc. children are here. What will happen
had a message saying that I had nearly Eventually, Barry arrives home at 6ish. to our routine without them? Briony
filled my storage and if I don’t do Meanwhile, Gilly and I have the bridge eventually lands in Mombasa – 30
something I will not be able to receive session from hell. I start by discarding people on a plane designed for 600,
any emails). Then I guess I need to do carelessly on a hand where I could have and the first incoming flight since
a bit of tidying. When Briony comes sworn dummy had had four diamonds the Lockdown. She says she felt like
home we have an enjoyable couple of when it actually had only three, so I let a celebrity as there were so many
hours cooking and preparing food through a ridiculous overtrick. Then paparazzi there to greet the plane.
for the evening: roast chicken dinner we take it in turns to do something Anyway, she is in good spirits – quite a
with all the trimmings, followed by stupid, and even when we don't do relief. With Toby gone too, it feels like
French apple tart. Barry, Margaret something stupid, our results do not the end of an era. n

Page 34 BRIDGE October 2020


BRIDGE is ceasing publication.
The last issue will be number 216 - December 2020.
See page 5 for more information.

Julian Pottage Answers Your Bridge Questions

Can A Michaels Cue Bid


Be Very Strong?

Q
The following players made a bid to bid one of the majors with a to say ‘provided sufficient
very distributional show their 2-suiter decent hand) trumps are available
deal occurred at a 3. At our table the 4
Expecting North to hold a weak for ruffing purposes’.
recent online club session bidding went: hand, South settles for game. For example:
and we would, please, like There is a case for bidding 4♦,
to have your observations West North East South though at this vulnerability
on how it should be bid. 1♠ 2♥ Pass 4NT1 South is expecting to buy the ♠ A Q 8 4
Pass 5♠2 Pass 6♥ contract in 4♥. ♥ 8
All Pass 5
With significant extra shape ♦ A Q 5 3
Dealer West. E/W Game. 1
RKCB West makes the unusual ♣ 9 7 4 3
♠ K Q 10 6 5 3 2
Two key cards plus the ♥Q (or, decision to sacrifice vulnerable
♥ A K 9 6 5 2 as here, enough hearts to think against not in 5♣, a contract
♦ 5 the partnership has a 10-card fit) that would in fact make. If spades were trumps,
♣ Void 6
Holding the strong variety most players would say
♠ A J 9 7 2 ♠ 8 On the lead of the club ace North must do something. Just this is a 6 loser hand.
N
♥ J W E
♥ 4 3 all 13 tricks were taken as all in case South has both pointed But could two trumps be
♦ 6 S ♦ 9 8 4 2 of North’s spades went away suit aces North cue bids 6♣. spared for ruffing hearts?
♣ K J 10 8 6 2 ♣ A 9 7 5 4 3 on dummy’s diamonds. 7
Shows first-round diamond Many years ago I came
♠ 4 4. Should West open 1♣ control but denies the ability to across a description of
♥ Q 10 8 7 and if so is it reasonable bid a grand slam. the Australian One Club
♦ A K Q J 10 7 3 for East/West to find System. In this system it was
♣ Q their 7♣ sacrifice? At any other vulnerability assumed that the first three
Steve Fieldhouse by email. the auction might end in trumps (if declarer) or the first
7♣ doubled. As it is East- two trumps (if responder)

A
The bidding was different On a wildly West need to get out for were unavailable for ruffing
at all 11 tables and only distributional deal 3 off in 7♣ doubled if the shortages in side suits. In this
two North/South pairs you would expect field is bidding and making case the hand above would
reached the laydown small a variety of results. small slam as North-South be a 6 loser hand if held
slam in hearts over which Here is a possible auction: and for 1 off if the field is by responder but a 7 loser
the opponents have an stopping in game. West hand if held by declarer.
excellent sacrifice in 7♣. West North East South also hopes to defeat 6♥ via Keith Jackson,
The various West players 1♣1 2♣2 4♣3 4♥4 the ♠A and a spade ruff. Cheam.
either passed or opened 5♣5 6♣6 Pass 6♦7

A
1♣ or 1♠; 1♣ appears Pass 6♥ All Pass ♣♦♥♠ The losing trick
preferable to me. 1
West has no reason not to count slightly

Q
1. At the two tables where open the longest suit. In the In the LTC, xx, x, overvalues 4-4-4-1
West passed, both Norths olden days people opened 1♣ and a void are hands. If you open 1♦
opened in hearts rather with 5-5 in the black suits. counted as 2, 1 and partner responds 1♠,
than with the correct 1♠. 2
Michaels, both majors (at least and 0 losers respectively you would definitely raise
2. When West opened 5-5), weak or strong provided there is a trump to 2♠ rather than 3♠.
1♣, none of the North 3
Pre-emptive (East would cue fit. Would it not be better u

BRIDGE October 2020 Page 35


t For the LTC to produce find it excellent in finding a The opening lead was a club. contract, West might well
the correct number of fit in both minors and majors. 3♥ by South made eleven lead a diamond, making
expected tricks your side Bruce McDougall, tricks, by two pairs. life easier for declarer.
generally needs a 9-card Co Durham. Should they have bid 4♥?
fit, so typically the player 1NT by North made ten ♣♦♥♠

A
who bids the suit first will We recommend tricks, by four pairs.

Q
have at least one more using transfers in 1NT by North made I have a bidding
trump than the player who response to a 2NT seven tricks, once. query. My
raises, so yes there is a logic opening, so a hand with five 3NT by South was one partner opened
in valuing declarer and spades but not four hearts light, by four pairs. 1♠; what should my
dummy differently. For using would respond 3♥, not 3♣. Could North make response be please?
the LTC, a 4-card trump Bidding 3♣ and then 3♠ eleven tricks in hearts?
holding is generally adequate would not therefore show Could the partnership
if partner has shown a five spades – a golden reach 4♥? ♠ J 2
5-card suit but might not be principle of bidding systems Alex Mathers, ♥ A J 5
enough if partner has only is that different sequences Northallerton. ♦ K Q 6
four, as could easily be the show different hands. ♣ A K J 8 2

A
case after a 1♦-1♠ start. If you are playing 2NT-3♣- With the ♠Q in the
If the auction in fact started 3♥-3NT as promising four East hand, declarer
1♦-(2♣)-2♠ then you place spades, you do not need 3♠ has two sure spade We do not play weak jump
partner with at least five as natural with four spades losers in a heart contract. shifts or Bergen raises
spades (a hand with only or with five spades; you With the ♥K and ♦Q both or anything like that.
four spades would make would need to agree it with onside, it is possible to avoid Elizabeth Kempshall,
a negative double.) In this your partner but the most any losers in the red suits Hove, West Sussex.
case, your hand would be helpful meaning might be a by taking all the available

A
worth a jump to 4♠ (a 4♥ hand with heart support too finesses and discarding Given that you play
splinter would perhaps be good for a simple raise to 4♥. a diamond on the ♣A. strong jump shifts,
doing a bit too much). Declarer might try ruffing you could start
♣♦♥♠ the third round of diamonds with 3♣ and rebid 3NT.
♣♦♥♠ in dummy, which only costs That would give a good

Q
On the deal a trump trick if West has general impression of the

Q
I was puzzled by below North in exactly three hearts including hand, though I would say
Andrew Kambites’ our room made the king (the actual layout) you would normally be in
answer to the ten tricks for a shared top. – if West has the doubleton the 16-18 range for that.
Slam Bidding Part 1 quiz king or East holds the king, If you start with 2♣ and
in BRIDGE 212, p35, Hand shortening North’s trumps partner rebids 2♠, you will
1B. He must play a different Dealer West. Game All. makes no difference. Perhaps be a bit too strong to rebid
version of Stayman to that ♠ K J 10 the right play is ruff a club 3NT and so need to invent
which I have used and ♥ A 10 7 at trick two and tackle a rebid, a 3♦ reverse for
taught for many years. He ♦ K 8 trumps at once, leading the example. Then if partner
states “If partner shows four ♣ A J 9 6 3 queen early, to discover the continues with either 3NT or
hearts you will bid 3♠ to see ♠ A 8 6 ♠ Q 5 4 position (and reducing the a fourth suit 3♥ you could
N
if he is 4-4 in the majors”. ♥ K 8 6 W E ♥ 5 3 chance of an adverse spade make an invitational 4NT bid.
For me this would show ♦ 10 7 6 5 S ♦ Q 9 3 ruff). If East (holding K-x-x)
that you hold five spades in ♣ Q 4 2 ♣ K 10 8 7 5 manages to duck the first ♣♦♥♠
your hand, and are looking ♠ 9 7 3 2 trump smoothly and wins

Q
for a 5-3 fit. I would rebid ♥ Q J 9 4 2 the second round and exits We had an
3NT instead and partner ♦ A J 4 2 with a third round, you still interesting
then knows that hearts ♣ Void have the diamond finesse bidding sequence
was not my suit and I must in reserve. Of course, with when, as West, I held this
therefore be holding four 24 HCP between the two uninspiring collection:
spades. With both majors West North East South hands, some pairs will not be
opener can, if he wishes, Pass 1♣1 Pass 2♦2 in game (as the results you
bid 4♠ depending on the Pass 2♥ Pass 3♥ quote confirm) and declarer ♠ Q 8 7 4
strength or weaknesses of Pass 4♥ All Pass will be reasonably happy ♥ 9 6 4 2
the other suits. Incidentally, 1
Strong, artificial (Precision just with 10 tricks. Ruffing ♦ 8 5 4
my partner and I do play style). a club could result in losing ♣ 10 2
the “old fashioned Baron 2
Transfer, promising at least five control if hearts are 4-1.
convention over 2NT” and hearts. If South declares a heart Continued on page 38... u

Page 36 BRIDGE October 2020


BERNARD  Answers to Sally Brock’s
MAGEE’S
INTERACTIVE
Simple Take-Out Doubles
TUTORIAL CD Part 4 Quiz, on page 19
ADVANCED What do you bid on the following hands with the auction given?
DECLARER The vulnerability and scoring method should not affect your answers.

PLAY get a decent penalty with


nothing making your way.
Hand 1 Hand 2 Hand 4 If partner had opened 2♥
♠ K 7 6 2 ♠ A Q 10 7 6 you would have settled for
♥ A Q 4 ♥ 7 6 2 an invitational bid in spades
♦ 7 6 3 2 ♦ A Q 6 5 (start with 2NT and then bid
♣ A 10 ♣ 4 3♠), but you are too strong
for a mere 3♠ after the 3♥
opening. If you bid 3♠, your
MAC
partner will place you with at
or Windows West North East South
most 7 HCP and possibly only
2♥ Dbl Pass a 4-card spade suit. Here you
? have a stronger hand with
a robust 5-card spade suit,
Hand 1 Bid 2NT, Lebensohl. Then, so you should jump to 4♠.
when partner bids 3♣ or
3♦, bid 3♥. This shows four
spades and a heart stopper,
Hand 5 Hand 6
so partner can choose
which game he prefers. ♠ 7 ♠ 8
Hand 2 Bid 3♠. This is forcing ♥ A Q 4 3 ♥ A Q 10 6 5
Bernard develops your (remember, you would bid ♦ K 10 5 4 ♦ K 10 6 5 4
advanced declarer play 2NT first if you had only ♣ J 10 3 2 ♣ 7 3
technique in the course of invitational values with five
spades). Even if partner
ten exercises and 120 new
is minimum you want to
complete deals. play in game, but if he
West North East South
has extras a slam is quite 4♠ Dbl Pass
l Overtricks in Notrump possible. If he has extra ?
Contracts values he will cue-bid.
Hand 5 A double of 4♠ should
l Overtricks in Suit be a strong no-trump or
Contracts
£81 Hand 3 Hand 4
thereabouts, and if partner
had opened one of those
l Endplay
£61
♠ Q 3 ♠ K Q 10 6 5 you certainly would not want
l Avoidance ♥ J 10 5 4 ♥ A 5 4 to be driving to the five level
♦ A Q 3 ♦ 10 5 on your 4-4-4-1 shape. Pass
l Wrong Contract
♣ 7 6 3 2 ♣ J 8 7 and hope for the best. Of
l Squeezes course, partner’s double
might not be so balanced
l Counting the Hand (he has to do something
West North East South
l Trump Trouble with any strong hand), so it
3♥ Dbl Pass could be right to bid, but the
l Doubled Contracts ? percentages favour a pass.
l Safety Plays Hand 6 Here with two good suits you
Hand 3 Pass. You don’t need to have should remove the double.
Operating system requirements:
Windows or Mac OS 10.08 -10.14 a very strong trump holding A 4NT bid on this auction
to pass a take-out double of shows two possible places
a three level opening. You to play. Partner should bid
Mr Bridge Limited don’t have enough to think his better minor. If he bids
3NT would make but you 5♦ you will pass, but if bids
( 01483 489961 should have good chances 5♣ you will continue with
www.mrbridge.co.uk/shop of beating 3♥. You may even 5♦ showing both red suits.

BRIDGE October 2020 Page 37


t ...Continued from page 36. We were E/W and the
auction was as follows: Dealer West. N/S Game.
Mr Bridge
West North East South
1♣ Dbl 1♥ West North East South


♠ A K 9
♥ K Q J 10 9
Luxury
Pass 2♣ 2♦ 3♣
Pass Pass Dbl Pass
1♦ 2♥
Dbl Pass 3♦ All Pass


♦ J 10 6
♣ 8 4
Playing
?
At the other table the initial
N
W E
Cards
S
We play a double of a bidding was the same, but
3 level bid for penalties after the double, North ♠ 10 7 6 5 4
but with our opponents raised to 3♥. East then rebid ♥ 4
having found a fit I did 3♠, which West raised to ♦ K 4 3
not think I could pass. 4♠, the game making with ♣ Q J 10 3
Margaret Moonan by email. the favourable spade split.
5♦ looks to be the best

A
Partner’s initial contract. How should West North East South
double followed by we have bid it? 1♣ 1♥ 2♣ Pass
2♦ showed a strong Hugh Ball, Pass 2♥ All Pass
hand with diamonds. Eastbourne, East Sussex.
The second double shows My partner felt that I, as

A
more of the same with either This is a tricky one. South, should have doubled
extra strength or extra shape. Neither West nor East 2♣ to show I had spades.
Since with four spades did anything terribly Given the vulnerability
and a longer diamond wrong – both players were and the weakness of my
suit, partner would have maximum for their bidding. suit I did not think so. I
doubled 2♣ rather than bid The hands happened to fit would appreciate your
2♦, I do not think we have perfectly – no wasted values advice and comments.
a 4-4 spade fit. I therefore in hearts facing the singleton Simon Bloomfield,
bid 3♦. Although the ♠Q – which is why 5♦ was on. Hitchin, Hertfordshire.
could be a useful card, the West had already shown

A
doubleton club is unlikely to a reasonable hand by With only 6 HCP and
be working. I do not wish to doubling at the two level no fit, South does
punish partner for competing but could have had a not have enough to
by jumping to 4♦ – it would couple of points fewer. double 2♣. Normally with a
be inconsistent to pass over East might have bid 3♥ 5-3-3-2 shape North would
3♣ on the previous round rather than 3♦ because if reopen with a double; with
and jump to 4♦ now. West had a heart stopper hearts this good I quite
it would be possible to understand treating the suit
♣♦♥♠ envisage scoring six diamond as if it was a six-card suit.
tricks, two spades and It looks as if 2♥ and 2♠ Box of 72 - £72

Q
May I ask for your a heart to make 3NT. would both go one down. (36 red, 36 blue)
advice as to how
we should have
Knowing that heart
ruffs would be in the short
Defending 2♣ doubled
would appear to be the
Strip of 12 -
bid these hands at teams. trump hand, the 3♠ bid winning move, though I do £19.95
chosen at the other table not think South would leave (6 red, 6 blue)
certainly had merit. in the double anyway. When
♠ 10 7 5 opponents bid and raise a Includes
♥ K 8 6 4 ♣♦♥♠ suit and you hold four cards postage to UK
♦ 9 6 5 2 in it, partner usually has a
addresses.
Q
♣ A 7 The following hands singleton. Furthermore South
♠ Q 9 6 2 ♠ A K 4 came up in a BBO has five spades and would
♥ 10 2 N ♥ 5 matchpoint game: expect North to have some
Cards are not
W E
♦ K J 3 S ♦ A Q 10 8 7 4 support. n barcoded and do
♣ K Q 6 4 ♣ J 9 2 not have tuck-
♠ J 8 3 Email your questions (including boxes
♥ A Q J 9 7 3
your postal address) for Julian to:
♦ Void
julianpottage@mrbridge.co.uk
( 01483 489961
♣ 10 8 5 3
www.mrbridge.co.uk/shop

Page 38 BRIDGE October 2020


Answers to Julian Pottage’s 
Defence Quiz on page 9
1. ♠ J 10 8 4 Partner leads the ♦3. What is your plan? compete to 3♠ and yet would hardly pass
♥ K Since you can see all the diamond North’s 1♦ opening with ♠K-Q-x-x-x, it is
♦ K Q 10 honours and the ♦2, the ♦3 lead must be very likely that partner has K-x or Q-x of
♣ Q J 9 6 4 a singleton. This means you can win three trumps.
♠ K 6 5 ♠ 7 2 tricks in the suit. What is more, partner This being the case, you need to
N
♥ A 9 5 4 2 ♥ Q 8 7 6 will be able to make two club discards duck the first trick, playing the ♣9 to
W E
♦ 9 3 S ♦ A 7 6 2 on your winners, perhaps setting up a encourage. You must then duck the first
♣ K 10 5 ♣ 7 3 2 ruffing opportunity unless declarer has a round of trumps, allowing partner to
♠ A Q 9 3 singleton or void in clubs. win. This way, when you get in with the
♥ J 10 3 You should win with the ♦10 (or capture ♣A, you can play a third club, enabling
♦ J 8 5 4 the ♦J with the ♦Q) and cash two more partner to ruff with a low trump.
♣ A 8 winners in the suit. West discards two
clubs and you switch to clubs. Declarer
cannot have all the high spades and the 4. ♠ K 9 5
West North East South ♣A yet pass as dealer, so partner will be ♥ K J 6
1NT1 able to get in, give you the lead with the ♦ Q J 8 7 2
Pass 2♣ Pass 2♠
1
♥A and score a club ruff for one down. ♣ K 10
Pass 4♠ All Pass ♠ Q 8 2 ♠ J 6
N
1
12-14 and Stayman ♥ 9 4 ♥ A Q 10 8 3
W E
3. ♠ J 10 8 4 ♦ A 10 6 3 S ♦ 5 4
Partner leads the ♦9. What is your plan? ♥ K ♣ J 5 4 3 ♣ A 9 7 2
You read the ♦9 as a short-suit lead. ♦ A K 10 7 4 ♠ A 10 7 4 3
It is far more likely to be a doubleton ♣ K Q J ♥ 7 5 2
than a singleton partly because there ♠ K 6 ♠ A 7 ♦ K 9
N
are many more doubleton holdings and ♥ A Q 9 6 2 ♥ 8 7 5 4 ♣ Q 8 6
W E
partly because South might have opened ♦ J 9 3 2 S ♦ Q 6
1♦ with a 4-2-5-2 shape. Lacking an ♣ 10 3 ♣ A 9 8 5 4
outside entry, you should duck, playing ♠ Q 9 5 3 2 West North East South
the ♦7 to encourage. A diamond ruff ♥ J 10 3 1NT1 2♥ 2♠
later will enable you to beat the contract ♦ 8 5 All Pass
by two tricks. ♣ 7 6 2 1
12-14

Partner leads the ♥9, covered by the ♥J.


2. ♠ 10 7 4 West North East South What is your plan?
♥ K Q Pass 1♦ Pass Pass You can tell the ♥9 is top of a doubleton
♦ J 8 6 1♥ Dbl 3♥ 3♠ (or possibly a singleton), which means
♣ K Q 10 8 4 All Pass you could take the ♥Q, cash the ♥A and
♠ K 8 5 ♠ J 3 give partner a heart ruff. The trouble
N
♥ 9 7 6 5 3 2
W E
♥ AJ Partner leads the ♣10. What is your plan? is that the ruff will quite often cost your
♦ 3 S ♦ A K Q 10 7 2 You read the lead as from a shortage. side a trump trick. If partner has Q-x-x, a
♣ J 7 5 ♣ 9 6 2 Since it looks as if you have the ♠A as trump trick would be coming without the
♠ A Q 9 6 2 an entry, it might seem that you can ruff but not after it. If partner has A-10-x,
♥ 10 8 4 afford to take the ♣A at once and return two trump tricks would be coming without
♦ 9 5 4 the suit in case partner has a singleton. the ruff but only one after it.
♣ A 3 The problem is that if the ten is top of Your hearts are good enough to score
a doubleton you may not get to give three winners without any ruffing so long
partner a productive ruff. The only way as the next heart lead comes from the
West North East South to get in will be to go up with the ♠A on other side of the table. After taking the
Pass the first round and play a third club. As ♥Q, switch to the ♦5. Partner should
Pass 1♣ 1♦ 1♠ you can see, partner would be ruffing deduce from your failure to continue
Pass Pass 2♦ 2♠ with the ♠K. hearts that you have the ♥10 and so will
All Pass Since South surely holds five spades to take the ♦A and revert to hearts. n

BRIDGE October 2020 Page 39


READERS’ If subscribers don’t like it
they can cancel at any time.
On Bernard’s site there
are, all included, one paid
is possibly a lost cause to
bernardmageebridge.com
but if he keeps watching
he will find that large

LETTERS
for 45-minute live seminar numbers of subscribers
per week, one free 45-minute are proving him wrong.
seminar (which non-
subscribers can continue to
watch), per week, one “over
the shoulder” live teams
GET A LIFE from Andrew Robson’s match which viewers can
One of your many attributes and so it should be – mine watch and hear Bernard’s
is the publishing of criticism was different to both of running commentary. There
of yourself, your products them. ‘Vive la difference’. are also, each month, on the
and your team. This serves to Apparently Gavin has site, two pre-recorded topic Will Parsons by email.
demonstrate that you really 160 passwords; well that seminars (45 minutes each),
care about your customers tells us everything. Come on 16 set hands, 4 bidding GOOD RIDDANCE
and have enough self esteem Gavin, ‘get a life’ and leave quizzes, 4 lead quizzes, and Gavin Wilson should see
to accept the criticism. Bernard to get on with his. 4 play quizzes plus forums the Bernard Magee Bridge
Critical letters do not On reflection, I’m where subscribers can ask website as a continuation/
have to be constructive but beginning to wonder if each other, and experts, any development of Mr Bridge,
they should be reasonable Gavin’s letter was a spoof bridge-related questions they undermined as it has
and proportionate. The or maybe he wrote after want. All previous seminars been by Covid-19.
rant against Bernard by taking a refreshment? and quizzes are archived The subscribers to this
Gavin Wilson in BRIDGE Ian Dalziel by email. and available making it very website are in the main
212 was neither, and should good value for money. There those who have for many
not have been printed. YOU’RE WRONG is also a private Facebook years enjoyed friendly
Gavin is clearly intelligent I was really disappointed to group for members and holiday bridge with
and articulate but his open the August edition of plenty of opportunities to play Mr Bridge, hosted by
arguments are absurd BRIDGE to find, on the very with other BMB members on Bernard Magee and others,
and Bernard is right not to first page, a long diatribe the free BBO site. Members catering for those who are
dignify them with a response. from Gavin Wilson strongly get much, much more for single and travelling alone.
I have never met Bernard criticising Bernard Magee their money than on Andrew Bernard will continue to
but everyone I know speaks Bridge. It runs to almost four Robson’s site which foster the care and attention
very highly of him and say columns, which I think makes Mr Wilson is so keen to extol. for his clients, and will listen
he is a thoroughly decent it the longest reader’s letter Mr Wilson is writing as to their views, to give value
and trustworthy man. He you have ever printed. Why? if Bernard has no right to for money as he always has.
must have been devastated In your July issue you say try and make a living, even He has made it very clear
when Covid put his whole how impressed you are with though bridge has been that anyone who does not
career on hold and he what Bernard has achieved, his career for more than wish to continue can leave
deserves great credit for that you are encouraging 25 years. It appears from with no further payment.
getting his online teaching him and that you encourage his letter that he was happy We will all continue to enjoy
off the ground so quickly. your readers to support to watch three times a week the family atmosphere and
He has to make an income him, and then you print while it was free but he you are welcome to take
from it and rightly so. that outrageous letter. begrudges a monthly cost your views to another place
With his talent I’m sure Just about every point which is, I am sure, less than that suits your needs.
Bernard could make a lot Mr Wilson makes is wrong. the monthly table money
more income in some other He writes that readers are he used to play for one
activity like IT, and thus making a ‘leap of faith’ in session of club bridge a week
have lots more free time, signing up at £13.99 per before lockdown started.
but he believes, as we all month with no knowledge He also makes a very strange
do, that bridge can enhance of what the website will look complaint about the number
people’s lives and he puts like. In fact Bernard had of ladies who take part.
that far above money. delivered 39 free seminars Doesn’t he understand the Helen and Crombie McNeil,
When Gavin says that in advance, giving a good age profile of most bridge Eastbourne, East Sussex.
£11.99 is a lot of money, feel of the style of offering, players? There are more
what planet is he on? and he had provided a ladies than men around in This correspondance is
He states that Bernard’s description on the ‘taster’ the bridge-playing sector. now considered closed.
teaching is slightly different page of the new site. I can see that Mr Wilson Continued on page 42... u

Page 40 BRIDGE October 2020


Answers to David Huggett’s 
Play Quiz on page 9
You are declarer in 3NT after East has will win with the king and return a heart
1. ♠ K 8 6 5 overcalled in spades. West leads the ♠7. setting up two potential losers for you in
♥ K J 7 How do you plan the play? that suit. Then if the diamond finesse fails
♦ 7 6 4 2 It may be easy to make an unwarranted you will have four losers.Much better is
♣ K 6 assumption on the hand, given that East to play the ace of clubs at trick one, draw
♠ 9 4 ♠ 3 has waded into the bidding. You might trumps (preferably ending in dummy),
N
♥ Q 8 5 4 W E
♥ A 10 9 3 be forgiven for thinking that East was and knock out the king of diamonds.
♦ K Q 10 3 S ♦ 9 8 5 bound to hold the ace of diamonds, in Assuming you have left yourself a spade
♣ A 9 4 ♣ Q 10 8 7 2 which case the contract was in no danger entry to dummy, you expect to make
♠ A Q J 10 7 2 at all; of course, these days overcalls are five spades, one heart, at least three
♥ 6 2 often paper-thin and a little thought diamonds and a club.
♦ A J should show that the position of that
♣ J 5 3 particular ace is immaterial.
Accordingly, play the queen of spades 4. ♠ 6 5
from dummy at trick one! ♥ J 8 7
You open 1♠ fourth in hand and find What can East do? ♦ Q J 10 5 4
yourself in 4♠. West leads the ♦K. How He has to capture this otherwise ♣ A J 2
do you plan the play? declarer will make two tricks in the suit ♠ Q 10 8 7 3 ♠ J 9 4 2
N
You are clearly going to be lucky but equally he cannot afford to continue ♥ Q 5 4 W E ♥ K 9 6 3
if the game makes and the king of the suit because declarer can let the ♦ A 7 2 S ♦ K 3
diamonds lead sets up a loser in that trick run round to the ten. Whatever he ♣ 6 4 ♣ 10 9 7
suit immediately. However there are still switches to then allows declarer plenty of ♠ A K
hopes of making six spades, a diamond, time to knock out the ace of diamonds. ♥ A 10 2
a club ruff and hopefully a trick in ♦ 9 8 6
both clubs and hearts with some lucky ♣ K Q 8 5 3
guesswork perhaps. At the table declarer 3. ♠ A 9 4 2
drew trumps and led the diamond jack ♥ 6 4 3
but West won with the queen and played ♦ Q 10 9 4 You are declarer in 3NT. West leads the
a low heart and declarer was at the ♣ A 8 ♠7. How do you plan the play?
crossroads. But was he really? For the ♠ J ♠ 7 5 3 It would be nice to play on diamonds
contract to stand any chance at all West ♥ K J 10 9 8 N ♥ Q 2 to knock out the ace and king but that
W E
had to hold the ace of clubs which meant ♦ K 8 5 3 S ♦ 7 6 line doesn’t stand a chance as you are a
he couldn’t have the ace of hearts as well ♣ 9 6 3 ♣ K J 10 5 4 2 tempo behind the defenders. They would
or he wouldn’t have passed as dealer, ♠ K Q 10 8 6 simply win the first diamond and play
making the play of the jack from dummy ♥ A 7 5 another spade, setting up the suit.
the only plausible action. ♦ A J 2 No, the extra trick you need can only
♣ Q 7 come from hearts and for that to happen
you need a bit of luck. Cross to dummy
2. ♠ Q 10 2 with a club at trick two and lead the heart
♥ K 7 4 2 You are declarer in 4♠. West leads the jack.
♦ Q 3 ♣6. How do you plan the play? If East ducks, you let West win and later
♣ K 9 8 2 An awful lot of the time a contract can finesse against East for the remaining
♠ 7 6 ♠ A J 9 8 3 fail because of insufficient thought going honour. More likely East will cover but
♥ 10 9 8 5 N ♥ Q J 3 into the play at trick one, and you have then you win with the ace and simply
W E
♦ A 7 5 4 S ♦ 8 6 to be careful that that does not happen reenter dummy with a club and lead the
♣ J 10 3 ♣ Q 7 5 here. heart eight, finessing against the nine if
♠ K 5 4 Just suppose you think that you have East plays low.
♥ A 6 been given the opportunity of making You might argue that East could have
♦ K J 10 9 2 a second club trick and play low from both missing heart honours without the
♣ A 6 4 dummy at trick one. On a good day the nine but he is twice as likely to have K-9-x
queen will score, but on a bad day East or Q-9-x than K-Q-x. n

BRIDGE October 2020 Page 41


READERS’ LETTERS UPDATE FROM THE
NEW OWNER OF
have this luxury. So we are
happy to provide information
continued... A. L. FLEMING
I am fully aware that a large
and assistance to those
looking for Perspex screens
number of club players are and produce an alternative
t ...Continued from page 40. up publishing your montly satisfying their bridge needs for the majority of clubs.
magasine BRIDGE. It was by playing on the various So, to the alternative, which
THANK YOU an essential read for anyone on-line sites but a far larger is a table screen system, that
FROM ERIN interested in bridge. Even a number of players, often we have designed to sell for
I would like to thank Mr learner would get much help decried as social players just £18.50 per table. The
Bridge and all the staff for from it. Its discontinuance will are awaiting the return to system consists of four pieces
the immense amount of really be a hole in our lives as their clubs. These social of white corrugated card,
enjoyment I have got from it was avidly read from cover players are, without doubt each piece measures 1.30m
the magazine which I look to cover and with lockdown the backbone of a game that x 70cm but folds in three so
forward to receiving. it filled a big hole in our lives is competitive but above all 50 x 70cm. When erected,
I have found each (don’t under estimate that). a social occasion, that can using simple Velcro fasteners,
magazine informative and However, as older people only be achieved by seeing, they provide each player with
interesting. One only has we all have to step back at speaking and playing with an enclosed side of the table
to look at the magazine to times when health starts to friends. Whilst I admit that I and a clear window looking
see how much trouble one become a major issue and it have a commercial interest onto 50cm square space, in
has gone into producing it. does appear that from what I in clubs re-opening, I also the centre of the table, where
I would also like to thank have read in your magazine have an interest in “keeping the action takes place. The
the person who has gone that this might be the cause bridge alive”, a situation I screens can be erected and
to so much trouble to send of your decision. We have fear will not be achieved if taken down in a matter of a
my magazine each time never been on any of your bridge becomes a game few minutes and either stored
with the most amazing holidays but am sure that of graphic interpretations or transported with ease.
stamps on it. I have also those that have will be even and button pushing. We are also making a
enjoyed looking at each more heartbroken than us. I have had many enquiries similar system for chess,
stamp to see what is on it. Well, we are left with the from clubs that would resulting in a considerable
I am indeed sorry to choice of what to do with our seriously consider re-opening saving on the £50+, that
see it finish. I think one is credit balance. It is necessary if they could acquire Perspex has been quoted for a
not nearly old until one for us to make a decision on screens at a reasonable single Perspex shield.
gets to 86. So if Mr Bridge our credit balance of £19.46. price. Alas, the demand for The other issue, is one of
is not that age maybe I am sure you do not want Perspex at present, makes social distancing, between
a few more magazines to be left with 100’s of tea me ask, does it have a tables, to assist with this.
might be on the way. towels so please feel free to “golden lining”? I hear of We are also producing
I will treasure the email me later once the dust costs ranging from £50 - our own “pull out” screens
magazines I have and has settled to offer us tea £200 per table, which is size 1 x 2m. These are
continue to reread them towels and any other bridge ridiculous, especially when similar to advertising signs
over and over again. Over paraphernalia that you hold you know that the final result but a meter wide, rather
the last few years I have in stock. However, as a direct will be a heavy piece of than the standard 80cm,
been in hospital a few answer to your options, make equipment that is awkward forming a clear plastic
times. Along with the bare that a donation to Little Voice. to handle and very easy to screen, that are easy to
essentials the only thing I Yes, we have enjoyed your break. Due to this, I have put out and take away,
have brought with me are monthly magazine no end. decided that we wouldn’t they are lightweight and
my BRIDGE magazines. Absolutely great and sadly be doing our customers a compact when not in use.
Wishing Mr Bridge a irreplaceable. Good luck in service if we couldn’t offer an Finally, we are staging
very happy retirement. the future Mr Bridge; you will alternative. Perspex screens some Zoom conferences,
To all the staff I never know what pleasure it are good for the clubs that with Nicky Bainbridge from
wish you health and has given us mere mortals. can leave their tables and Rugby. We would be happy
happiness in your lives. Graham Stevens, screens in situ 24/7, but not to invite you to take part.
Above all keep Caterham, Surrey. for the majority who don’t We would also be happy
safe and healthy. to assist anyone who may
Meabh Gallgher, Dublin. Write to Mr Bridge at: Ryden Grange, Knaphill, have concerns or questions.
Surrey GU21 2TH or e-mail letters@mrbridge.co.uk
Do please contact me:
GIVE THE BALANCE E-mail correspondents are asked to include their name, terry@playbridgedealer4.com
TO LITTLE VOICE full postal address, telephone number and to send no Terry Collier,
attachments.
I am most sad that you A.L. Fleming’s Bridge Shop,
Letters may be edited for length and clarity.
feel it necessary to give Dunmow Road, Herts. n

Page 42 BRIDGE October 2020


BRIDGE is ceasing publication.
The last issue will be number 216 - December 2020.
See page 5 for more information.

Answers to Acol Two Bids And Strong


Bids In Benjamin Acol Quiz on page 19
1 Consider the following hands open 2♠, though 1♠ is a 2♦ and the quality of the
(i) How many playing tricks does the reasonable alternative. spade suit is not good
hand have? (vi) Open 2♣ intending to enough for an Acol 2♠ or a
(ii) How many points does the hand rebid 2♠ over a 2♦ negative Benji 2♣ with a 2♠ rebid.
have? response, or open 1♠. Hand 1E: (i) 8 PT – spades 8 PT
(iii) How many controls does the Hand 1B: (i) 9 PT – spades 5 PT & (ii) 10 points
hand have? (Ace = 2, King = 1) diamonds 4 PT (iii) 3 controls
(iv) Under EBU regulations are you (ii) 13 points (iv) It should be described
allowed to describe the hand as (iii) 3 controls as: ‘Either a strong hand
strong? (iv) With only 13 points or eight playing tricks’.
(v) At love all what would you open if you would need 5 controls (v) and (vi) Open 4♠.
you are playing Acol with strong to describe it as strong. It Pre-emptive. You have
twos? should be described as: very little defence.
(vi) At love all what would you open if ‘Either a strong hand or Hand 1F: (i) 8 PT – diamonds 8 PT
you are playing Benji? eight playing tricks’. (ii) 10 points
(v) You could open 2♠, (iii) 3 controls
though 4♠ is a reasonable (iv) It should be described
Hand 1A Hand 1B alternative. The point is as: ‘Either a strong hand
♠ A Q J 10 8 5 ♠ K Q J 10 6 5 that you don’t have much or eight playing tricks’.
♥ Void ♥ Void defence to an opposition (v) and (vi) Open a
♦ A K 6 5 4 ♦ A Q J 5 4 contract in hearts or clubs. gambling 3NT, showing
♣ 7 4 ♣ 7 4 (vi) Open 2♣ intending to a solid 7 or 8 card minor
rebid 2♠ over a 2♦ negative with very little outside.
response, or open 4♠.
Hand 1C Hand 1D Hand 1C: (i) 9 PT – spades 4.5 PT, 2 You are playing Benji as I have
♠ A K Q 5 4 ♠ K 6 4 3 2 hearts 2.5 PT, diamonds 2 PT described.
Suit bids forcing to suit
♥ A K 5 4 ♥ A J (ii) 23 points agreement after a Benji 2♣ opening
♦ A K 2 ♦ A K (iii) 9 controls and no values shown by responder.
♣ 3 ♣ A Q 5 4 (iv) It meets the requirements For bidding strong, balanced hands
to be described as ‘strong’. you have the following agreement:
(v) Open 2♣. Game forcing. 2NT opening bid = 20-22 points
Hand 1E Hand 1F (vi) Open 2♦, the strongest Benji 2♣ with a 2NT rebid over a 2♦
♠ A K Q J 7 6 5 2 ♠ 7 possible Benji bid, and negative = 23-24. Not forcing.
♥ 7 6 ♥ 7 6 rebid 2♠ over a negative Benji 2♦ with a 2NT rebid over a
♦ 9 4 ♦ A K Q J 7 6 5 2 2♥. This is game forcing. 2♥ negative = 25 upwards. Game
♣ 7 ♣ 9 4 Hand 1D: (i) 7 PT – spades 2 PT, forcing.
hearts 1 PT, diamonds: In Auctions 2A to 2F, is the last bid
2 PT, clubs: 2 PT forcing?
Hand 1A: (i) 9 PT – spades 5.5 PT & (ii) 21 points
diamonds 3.5 PT (iii) 8 controls Auction 2A Auction 2B Auction 2C
(ii) 14 points (iv) It meets the requirements West East West East West East
(iii) 5 controls to be described as ‘strong’. 2♦ 2♥ 2♣ 2♦ 2♣ 2♦
(iv) It meets the requirements (v) and (vi) Open 2NT. The 2♠ 2NT 2♠ 3♣ 2♥ 2NT
to be described as ‘strong’. hand isn’t strong enough 3♠ 3♠ 3♦ 3♥
(v) You could theoretically for an Acol 2♣ or a Benji u

BRIDGE October 2020 Page 43


Answers to Acol Two Bids And BERNARD 
Strong Bids In Benjamin Acol MAGEE’S
Quiz on page 19 INTERACTIVE
TUTORIAL CD
Continued...
ACOL BIDDING

t
Auction 2D Auction 2E Auction 2F Hand 3C Hand 3D
West East West East West East ♠ J 8 5 4 3 ♠ Q 6 4 MAC
2♣ 2♠ 2♣ 2♦ 2♣ 2♥ ♥ K J ♥ A K 4 or Windows
3♠ 2♥ 2♠ 2NT ♦ A K Q 3 ♦ K Q 7 6 5
3♠ ♣ A K ♣ A J
Auction 2A: 3♠ is forcing. A Benji 2♦
opening is game forcing.
Auction 2B: 3♠ is forcing. 2♦ was a Hand 3E Hand 3F
negative response but ♠ A Q 8 ♠ A Q 8
East didn’t give a second ♥ K Q 8 2 ♥ K Q J 9
negative of 2NT. 3♣ shows ♦ A 5 ♦ A K
4-7 points and a club ♣ A K J 10 ♣ A K J 10
suit, enough to make the Throughout 200 deals split
auction game forcing. into ten chapters, Bernard
Auction 2C: 3♥ is not forcing. East Hand 3A: Open 2NT. 19 points but evaluates your bids, praising
has given two negative the shape and intermediates the correct ones and
responses. 2♦ shows make it as good as discussing the wrong ones.
0-7 points. 2NT shows most 20 point hands.
0-3 points. Hearts Hand 3B: Open 2♣ and rebid 2NT l Opening Bids
and Responses
£66
are bid for the second over 2♦. This shows 23-24
time in the auction.
Auction 2D: 3♠ is forcing. East gave a
points. You have 25 points
but the poor shape and lack
l Slams and
Strong Openings
£46
positive response to 2♣. of intermediates suggest l Support for Partner
Auction 2E: 3♠ is forcing. East gave downgrading the hand.
l Pre-empting
a negative 2♦ response Hand 3C: Open 2NT. The spades
to 2♣ but then bid a are not remotely good l Overcalls
natural 2♠ rather than the enough to open 2♣ and l No-trump Openings
second negative of 2NT. rebid 2♠, which shows an and Responses
Auction 2F: 2NT is forcing. 2♣-2♦-2NT Acol two bid in spades. l Opener’s and
is not forcing but East’s 2♥ Hand 3D: Open 1♦. An average 19 Responder’s Rebids
is a positive response. points. The 5-card diamond
l Minors and Misfits
suit is promising but the
3 You are playing Benji. What is your hand has no intermediates. l Doubles
opening bid with these hands. If you Hand 3E: Open 2♣ and rebid l Competitive Auctions
open 2♣ or 2♦ what is your rebid 2NT over 2♦. This
after a negative response? shows 23-24 points.
Hand 3F: Open 2♦ and rebid 2NT Operating System requirements:
over 2♥. This is game forcing Windows or Mac OS 10.08 -10.14
Hand 3A Hand 3B showing upwards of 25
points.


♠ Q 10 9
♥ A K 10
♠ A Q 4
♥ K Q J
n
Mr Bridge Limited
♦ K Q J 6 5 ♦ A Q 5 2 ( 01483 489961
♣ A 10 ♣ A K 3 www.mrbridge.co.uk/shop

Page 44 BRIDGE October 2020


Answers to Bernard Magee’s 
BERNARD  Bidding Quizzes 1-3
MAGEE’S
INTERACTIVE on page 7
TUTORIAL CD

1. Dealer East. Love All. West North East South
DEFENCE ♠ 2
N
♠ A K 10 4 3 ?
♥ 7 2 W E ♥ K Q 6
♦ 4 3 S ♦ A 6 2 4♥. You have eight tricks in hearts, but
♣ K J 7 6 5 4 3 2 ♣ Q 9 this is not a ‘strong’ hand. To open with a
strong bid, you should have a hand that
can make some tricks in any contract –
MAC West North East South no-trumps, spades, hearts, etc.
or Windows
1♠ Pass When you have a hand that will make
? a large number of tricks with one suit as
trumps, but very few against other suits
1NT. Just 4 high card points, but an or as dummy, then you are looking at
8-card suit – what on earth should you a hand that should pre-empt. Bid high
do? and fast to stop the opponents from
You would like to play with clubs as encroaching on the auction.
trumps and choose the right level to play With 8 solid hearts, 4♥ is the correct
at. If your partner has 12-16 points, a bid – you will make the game if your
partscore is probably best, whilst if he is partner can supply two quick tricks, but
strong, you might be able to make 5♣. at the same time you will make it difficult
Bernard develops your Any club bid you make will not paint for your opponents to compete for the
defence in the course of ten the right picture: 2♣ shows 10+ HCP, deal.
introductory exercises and 3♣ shows 16+ and 4♣ is used as a Here, 4♥ will go one off, but your
120 complete deals. splinter. This is perhaps the rare example opponents can surely make 4♠.
of needing 4♣ naturally, but even if 4♣
l Lead vs was natural – it might take you too high.
No-trump Contracts You could pass, but surely 1♠ is 3. Dealer East. Game All.
l Lead vs unlikely to be the best contract from your ♠ 2 ♠ A K 9 7 5 4 3
N
Suit Contracts perspective. What you would like to do ♥ K Q W E ♥ 4 3 2
is show your weakness and your clubs. ♦ 4 3 S ♦ 6 2
l Partner of Leader
To do this, you have to start with a 1NT ♣ K 8 7 6 5 4 3 2 ♣ 9
vs No-trump Contracts response. This might seem crazy, but it
l Partner of Leader sets the tone for the rest of the auction.
vs Suit Contracts Your subsequent club bids will show very West North East South
l Count Signals long clubs and extreme weakness. 3♠ Pass

l Attitude Signals
£76 If your partner passes your 1NT ?

£56
response, you will go down in 50s, when
l Discarding surely your opponents have something Pass. As a general rule, if your partner
l Defensive Plan good on in hearts. pre-empts in front of you, you cede to
Here, after your 1NT response your his suit. The only exceptions are when
l Stopping Declarer
partner might even jump to 3NT, over you are very strong and might want to
l Counting the Hand which you could jump to 5♣ – 8 clubs discuss the partnership’s options further.
opposite 18-19 points gives a reasonable This means that a change of suit opposite
chance for game. a pre-emptive opening is a forcing bid.
Operating System requirements: What this means is that you cannot
Windows or Mac OS 10.08 -10.14 simply bid 4♣ and expect to be left there.
2. Dealer West. Game All. With two weak hands, you should pass
partner’s 3♠ and hope he can muster
Mr Bridge Limited

♠ 3
♥ A K Q J 9 8 7 2 N
♠ 9 8 7
♥ 5 4 enough tricks. Here, 3♠ is likely to go a
( 01483 489961 ♦ 4 2 W E ♦ K Q 9 8 couple off – even if it is doubled that is
S
www.mrbridge.co.uk/shop ♣ 8 2 ♣ Q J 5 4 just -500 which the opponents can surely
beat by bidding and making 4♥ (620). n

BRIDGE October 2020 Page 45


The Diaries of Wendy Wensum
Episode 102:
Any Questions Partner?

A
few autumns ago, Spouse Riverside. I was playing with Spouse. Spouse was sitting South, a pint of real
and I planted a pear tree in I can usually bribe him to partner ale close at hand. Unlike his reaction
our garden. There it stood me when necessary by reminding to wet feet, he could not be accused of
throughout winter, spring and him of the excellent range of real wimpish behaviour on this occasion
summer without any signs of life. We ales available. It always works unless as he opened 1♥. It was true his hand
christened it the stick, proudly took there is a special football or rugby technically contained more than 19
a photo of it and showed the latter match live on TV; our teammates opening points, but I suspect many
to the nursery who happily replaced were Kate and Jo. Computer dealt- Riverside players would open a weak
it. Spouse planted the new sapling boards can be relied on to give the odd 2♥ instead. West passed and I bid
last autumn and on his return to the freak distributions. This deal was no 2NT, agreeing hearts as trumps and
house, removed his boots and left them exception. forcing to at least 3♥. East now came
forgotten on the lawn. Overnight the in with 4♠. Spouse bid 5♦ and West
rain was excellent for the tree, but not came to life with 5♠. Holding a spade
so good for the boots. Not surprisingly Dealer South. E/W Game. singleton, I ventured 6♥, which ended
in the morning they were full of ♠ 7 the auction.
water. ‘What should I do? How will I ♥ K Q 5 2
get them dry?’ he moaned. Crushed ♦ A Q 10 6 4 West North East South
up newspapers were stuffed into the ♣ A 9 5 Wendy Spouse
boots, which then spent a day or so ♠ A 6 5 4 ♠ K Q J 10 9 8 3 2 1♥
in the airing cupboard. They were ♥ 10 7 4 N ♥ Void Pass 2NT 4♠ 5♦
1
W E
soon restored to their former glory. ♦ K S ♦ 5 5♠ 6♥ All Pass
Honestly sometimes Spouse can be a ♣ K J 8 4 2 ♣ 10 7 6 3 1
Forcing with at least a four-card heart suit
bit of a wimp. The new tree is thriving ♠ Void
and a fine addition to our garden. ♥ A J 9 8 6 3 ‘Any questions?’ asked East, laying
It was teams of four at the Riverside. ♦ J 9 8 7 3 2 a card face down on the table. ‘Yes, I
Unusually Millie was an absentee, ♣ Q have,’ said West,’ Why are you asking
having been invited to play as a guest at me? It’s my lead, South is declarer.’
a neighbouring club as president of the During this banter, Spouse was busy

Page 46 BRIDGE October 2020


placing his cards down as dummy.
He suddenly stopped, looked up and Answers to Bernard Magee’s 
quickly replaced the cards in his
hand, simultaneously displaying an
embarrassed grin. His antics were
Bidding Quizzes 4-6
of no help to the defenders. West
formally went through the correct
procedure of asking, ‘Any questions
on page 7
partner?’ On receiving a negative
response, he displayed the ♠A. Spouse
won in hand, removed the opposing 5♣. Your partner has made a take-out
trumps and claimed all the tricks 4. Dealer West. N/S Game. double, whilst your opponents have
when the ♦K appeared from West. As ♠ 2 N ♠ Q J 10 9 found a spade fit. It looks highly likely
it was impossible to go wrong with ♥ 7 W E ♥ A K 3 that they can make a game in spades. At
the ♦K singleton onside, the grand in ♦ K 9 8 7 6 5 4 3 S ♦ 10 2 the same time, you surely have a big fit in
hearts is always making, whereas 7♦ ♣ J 4 3 ♣ A 9 8 7 clubs. Your partner has shown shortage
by South can go off on a heart lead. in spades and support for the other suits.
As we scored up at the interval, we You can imagine making 5♣ – if partner
found we had a good result on the West North East South has a singleton spade and a couple of
hand. South had opened a weak two in 4♦ Dbl Pass 4♠ aces. However, if 5♣ is going off there is
hearts. Kate as West passed. North’s ? no doubt that 4♠ will be making. So you
2NT was an enquiry regarding the should jump to 5♣, to make or to save.
strength and quality of South’s Pass. At favourable vulnerability you 5♣ goes one off, whilst the opponents
holding. Jo jumped to 4♠. South now chose to open 4♦ on this hand – with 8 can probably make 11 tricks in spades.
bid 5♦. When asked North was unsure cards that is a reasonable risk. However, By jumping to 5♣ you make it difficult for
of the meaning. Kate, with a near you have not managed to stop the them to judge what to do next.
opener, supported Jo by bidding 5♠. opponents from bidding a game. There
North thought for a long time before is then the thought that if they can make
bidding 6♥. Jo then sacrificed in 6♠. game (620) then you could bid 5♦ and if 6. Dealer West. Love All.
you make 8 tricks, that would be a great ♠ Q J 8 7 6 5 4 3 ♠ 2
West North East South sacrifice against their game (-500 for ♥ A K N ♥ J 8 6 5
W E
Kate Jo three off doubled). ♦ A 3 ♦ Q 8 7 4
S
2♥1 Of course, this is how things look from ♣ 6 ♣ A 9 3 2
Pass 2NT 4♠ 5♦
2
your side of the table – from your partner’s
5♠ 6♥ 6♠ Pass side of the table, he is thinking – ‘great
Pass Dbl All Pass pre-empt partner – you have managed West North East South
1
Weak, 5-9 HCP to get the opponents to bid to the wrong 1♠ Pass 1NT Pass
2
Ogust contract’. He is waiting to double 4♠. ?
You should only ever pre-empt once on
South led the ♥A ruffed in hand by Jo. a hand – after that things are left to your 4♠. You chose to open just 1♠ with this
The contract of 6♠ slipped one off as partner. On a really good day 5♦ might hand because you had good all-round
Jo conceded the two minor-suit aces actually make, but 400 does not score as strength, but not enough to suggest a
to North. well as 500 for defeating 4♠ doubled by strong 2-level opening. Your partner
At the end of the bridge we retired two tricks or maybe even 800 if you get has shown limited strength opposite, so
to our usual tavern. Even without a heart ruff. a slam is out of the question, but surely
Millie, we still managed to discuss the 4♠ is a reasonable assessment of your
apparent high frequency of freakish combined assets.
distributions in the era of computer 5. Dealer North. N/S Game. East could have a void in spades, but
generated hands. Jo’s theory is that in ♠ 8 7 6 ♠ J it is reasonable to hope for one spade at
N
the interest of reducing running time ♥ 7 W E ♥ A J 9 5 least and then you hope he has a trick
and saving memory space, the coding ♦ 4 S ♦ Q J 8 5 3 outside the suit for you. You have to bid
of the algorithm on a small machine is ♣ K 10 8 7 6 5 4 3 ♣ Q J 9 the game yourself – inviting partner will
simplified and as a result the quality get little response. 3♠, after all, might
of random number generation is less show just six spades, so he would pass
reliable. Perhaps some pear trees suffer West North East South quickly.
a similar fate caused by a random lack 1♠ Dbl 2NT* 4♠ has good chances – if you can avoid
of individual care leading to an equally ? three trump losers you should make, and
unsatisfactory outcome; witness our * spade raise with 10+ points this is with your partner having a poor
stick as an example. n hand. n

BRIDGE October 2020 Page 47


Julian Pottage Answers Your Frequently Asked Questions

When Is A New
Suit At The Three
Level Forcing?

1
When both members of the Auction C just forcing for one round, it is forcing
partnership are bidding, West North East South to game. In Auctions, B, C and D the
introducing a new suit at the three 1♠ 2♥ West hand is unlimited. In Auction A,
level is forcing unless a previous action 3♣ the East hand is unlimited.

2
makes this illogical. Now let us see some auctions when
When one player is bidding solo, Auction D a new suit is not forcing and examine
a new suit at the three level will West North East South the reasons why.
only be forcing if that player has 1NT Pass

3
already shown great strength. 2♦* Pass 2♥ Pass Auction E
When one partner has made a 3♣ West North East South
takeout double, a suit bid by the *5+ hearts 1♥ Pass
other partner is not forcing and a 1NT Pass 2♦ Pass
suit introduced by the doubler is only In Auction A, opener is showing at 3♣
forcing if the other partner has shown least 15 points and responder at least
some values. a good 9 points. No way would opener Auction F
want to have to jump to 4♣ to show West North East South
Let us examine the logic and see some a good hand with clubs. By playing Pass Pass 1NT Pass
examples of the above. 3♣ as forcing not only does this allow 2♦* Pass 2♥ Pass
When both members of the space for the auction to develop when 3♣
partnership are bidding, you are opener has hearts and clubs but it also * 5+ hearts
exchanging information, trying to allows the partnership to assign a
find the best contract. If one of you special meaning to a jump to 4♣. With In Auction E, West has denied the
introduces a new suit, this suggests a weaker opening hand with hearts values to make a two-over-one 2♣
you have yet to establish the correct and clubs, opener simply rebids 2♥. response while East has not promised
denomination and so want to leave Likewise in Auction B, responder anything more than a minimum
room to do so. In case you want to play would not want to have to jump to opening bid. West’s sequence shows a
in 3NT, you do not want to have to 4♣ to show a good hand with clubs weakish hand with a long club suit.
jump to the four level to create a force. as a second suit. With a weaker hand, Auction F looks suspiciously like
In case the deal is a misfit, likewise responder has the options of passing Auction D. Have you spotted the
you do not want to have to jump to 2♥ or rebidding 2NT. difference? This time West passed as
show your suit. In Auction C, West’s 3♣ is a non- dealer, limiting the hand. A passed
jump change-of-suit response, albeit hand cannot have the values to force
Auction A at a higher level than usual because of facing a weak no-trump opening.
West North East South the overcall. If, however, 1NT is strong then 3♣
1♥ Pass 2♦ Pass Auction D is similar to Auction is forcing – sorry this is a bit of an
3♣ B. Responder does not want to have anomaly.
to jump to show a secondary club
Auction B suit and a good hand; again, passing Auction G
West North East South 2♥ and rebidding 2NT are options West North East South
1♥ Pass available on weaker hands. 1♥ 1♠
2♦ Pass 2♥ Pass In all of Auctions A to D, West’s 2♥ Pass Pass 2♠
3♣ new suit bid at the three level is not 3♦

Page 48 BRIDGE October 2020


West
Auction H
North East South
Answers to Bernard Magee’s 

3♥
1♠ Dbl 2♠
Bidding Quizzes 7-9
In Auction G, East has announced that
game values are not present by passing
on page 7
2♥, so 3♦ cannot be a game try. Instead
it is competing in the auction and
offering an alternative place to play.
West will typically have three hearts
and six diamonds in a hand too weak 4♣. Your partner’s weak no-trump gets
to have bid 2♦ over 1♠. 7. Dealer North. N/S Game. doubled and you have an eight-card
In Auction H, the partnership has ♠ 4 N ♠ 9 7 6 5 2 minor. You are certainly not going to
not bid any suits previously, so hearts ♥ 8 2 W E ♥ A654 leave your partner in 1NT, but it is a
is the first suit rather than a new suit. ♦ 7 6 S ♦ Q 8 2 question of how many clubs to bid.
Assuming that that doubler has some ♣ A 10 9 7 6 5 4 3 ♣ 8 You have plenty of potential in clubs,
heart support, West would simply but at the same time, you can expect your
jump to 4♥ with a better hand or, if opponents to have a fit somewhere and
unsure of the correct denomination, West North East South possibly even the potential for game.
West could double 2♠. 2NT Pass 3NT So you want to make it difficult for your
? opponents to compete further in the
auction, but without risking too large
Auction I 4♣. On a lot of weak shapely hands, the a penalty. 4♣ seems the right level –
West North East South vulnerability is key. Here, your opponents bidding to the level of your fit – partner
1♠ are vulnerable and you are not, so you has at least two and you have eight,
2♥ 2♠ Pass Pass might consider a sacrifice – three off making ten in total, so bid to make ten
3♦ doubled will give you a profit against tricks. In the play you expect to make
their game. 7 club tricks and surely one from your
Your partner is not going to lead clubs partner.
Auction J against 3NT (and even if he does you Bidding 4♣ makes it difficult for your
West North East South have no side entry) so you cannot see opponents to reach 4♠.
2♣* 2♠ Pass Pass much chance of defeating their game.
3♦ How likely are you to make 7 tricks in
4♣? 9. Dealer West. Game All.
In Auction I, only one member of the You expect to have no more than 2 ♠ 4 ♠ A K 8
N
partnership is bidding, which means club losers, but you might also have just ♥ 8 2 W E
♥ Q J 10 4
that 3♦ is non forcing. On a hand with one loser, which is good enough (without ♦ 7 6 S ♦ A J 10 3 2
great strength, West might have made an outside trick) and it is reasonable to ♣ K 8 7 6 5 4 3 2 ♣ Q
a 2♠ Michaels cue bid on the previous hope partner might have one trick for
round (planning to bid again) or you.
doubled. It is a gamble, but a calculated one West North East South
In Auction J, if West’s 2♣ opening – even if they can make 4NT, they may 3♠
is Acol then the 3♦ rebid is forcing, be reluctant to bid it so they may find it ?
just as it would have been if North had difficult to get the game bonus.
passed and East had made a negative 4♣ is likely to go two or three off, for a Pass. Never pre-empt a pre-empt. What
response. reasonable gain against 3NT. does this phrase mean? You pre-empt
If, however, 2♣ is Benjamin (so not to disrupt potentially strong opponents,
the partnership’s strongest opening), so when an opponent indicates that he
3♦ is non forcing – East might have 8. Dealer East. Love All. is weak by pre-empting that rationale
a Yarborough and West has denied ♠ 4 ♠ Q 6 5 no longer applies. Your partner will
N
the ability to make a game forcing ♥ 8 2 W E
♥ KQ54 consequently assume you have a suitable
opening. ♦ 7 6 S ♦ A 10 3 2 hand if you overcall and he can therefore
Trying to remember every possible ♣ A 8 7 6 5 4 3 2 ♣ J 9 act accordingly. If you overcall 4♣, your
auction is difficult. The important partner will play you for a good hand –
question to ask yourself is whether he will raise to game or perhaps try 6NT!
both players have limited their hands. West North East South You pass and allow your partner to
Unless they both have, a new suit at 1NT Dbl bid, possibly 3NT, then you can choose
the three level is going to be forcing. n ? your bid: 4♣ might be enough. n

BRIDGE October 2020 Page 49


Letter From Overseas by John Barr
BERNARD 
MAGEE’S
INTERACTIVE
TUTORIAL CD
Quarantine
DECLARER
PLAY

K
itzbühel Bridge Club was West North East South
one of the first in Europe to Pass 4♣
start playing again after the 4♥ 5♣ Pass Pass
COVID-19 lockdown. Dbl All Pass
I confess that I was concerned about
MAC the health risks, especially with so West started with the ♥A and the
or Windows many elderly people playing bridge, ♥K, and then made the fatal mistake
but the club responded well to the of trying to cash the ♦A. My partner
challenge. Everyone must wear a ruffed this, drew trumps and threw his
face covering, with free masks and losing spades away on the established
subsidised visors available. There is a diamond tricks – thus making his
bottle of hand disinfectant on every contract. After cashing the top hearts,
table, and those players having to West knows that declarer has seven, or
move from table to table must take more likely eight, clubs to go with his
their own bidding box with them. The two or three hearts.
only downside for me is that, when If you consider the different holdings
Bernard develops your
faced with a difficult dummy play, I that declarer may have in spades and
declarer play technique in
sit back to plan the hand, then pick up diamonds, switching to the ♠K will
the course of ten my beer for a quick swig and crash the never give away the contract. Even
introductory exercises and drink into my visor. if declarer has ace to three and your
120 complete deals. Here is a hand from yesterday’s club spade switch costs a trick by helping to
l Suit Establishment
game where the defence should have set up the ♠J, it’s the second undertrick
considered putting a key defensive that you concede, not the contract.
in No-trumps
card into quarantine for a while. And if declarer is short in spades then
l Suit Establishment your ♦A is guaranteed to win a trick.
in Suits So at trick three you can put your ♦A
l Hold-ups
£76
Dealer East. E/W Game.
♠ J 8 4 2
into quarantine and switch to the ♠K,
safe in the knowledge that you will
l Ruffing for
Extra Tricks £56

♥ J 4
♦ K Q 4 3
win at least one more trick with the
♠Q or the ♦A.
l Entries in ♣ K J 8 Note that bidding 5♣ guaranteed us
No-trumps ♠ K Q 10 ♠ A 7 6 3 a good score on this hand. Worst case,
N
l Delaying Drawing Trumps ♥ A K Q 10 8 W E ♥ 7 6 2 if the opponents doubled us and then
l Using the Lead ♦ A 10 6 2 S ♦ J 9 8 7 5 defended perfectly, we would go two
♣ 3 ♣ 5 off and lose 300 for a middle score. If
l Trump Control ♠ 9 5 they failed to double, then even two
l Endplays & Avoidance ♥ 9 5 3 off for -100 would have been a top, and
l Using the Bidding ♦ Void the mis-defence which let the contract
Operating system requirements: ♣ A Q 10 9 7 6 4 2 through was just icing on the cake.
Windows or Mac OS 10.08 -10.14 It is often worth pushing the boat
out in competitive situations when you
Most pairs played in hearts at the are non-vulnerable against vulnerable
Mr Bridge Limited three or four level, making 10 or opponents. And even if it is actually
( 01483 489961 occasionally 11 tricks. The opponents the wrong thing to do, your opponents
www.mrbridge.co.uk/shop bid game against us and we pushed on may nevertheless bid one more and be
to 5♣: defeated. n

Page 50 BRIDGE October 2020


BERNARD  Answers to Bernard Magee’s 
MAGEE’S Bidding Quizzes 10-12
INTERACTIVE
TUTORIAL CD on page 7
ADVANCED
ACOL BIDDING
and now how do you get across that your
10. Dealer North. Game All. spades are so powerful?
♠ 4 3 2 N ♠ A K Q 9 8 Since 2♣ followed by 2♠ is forcing to
♥ 4 W E ♥ 9 7 game, you could rebid 2♠. However, you
S
♦ K 8 7 6 5 4 3 2 ♦ 9 want to tell your partner that spades are
♣ 7 ♣ A K 9 8 5 trumps whatever he holds. To do this
you jump to 3♠ – while this appears to
be an unnecessary use of bidding space,
West North East South you are doing it for good reason. Your
3♥ 4♥* Dbl 3♠ rebid says: ‘Partner, I do not want to
? discuss the trump suit – we are playing
* strong two-suited hand in spades – please show any useful high
card you have.’
4♠. Your partner has bid the opponents’ East cue bids 4♥ and you can then bid
suit – what should it mean? to the slam.
Throughout 200 deals, In this kind of auction, over a pre-
Bernard not only evaluates empt, it shows a two-suited hand, in this
your bids, praising the case not the minors because 4NT would 12. Dealer East. Game All.
show that. ♠ Void ♠ A K Q J 10 9 4
correct ones and discussing N
Almost always you should choose to ♥ A 2 ♥ 6 3
the wrong ones, he also W E
play in one of your partner’s suits, even ♦ 7 6 5 S ♦ A K Q J
plays the hands with you.
with an 8-card suit of your own. If your ♣ J 8 7 6 5 4 3 2 ♣ Void
l The Basics £96 suit was stronger, then maybe you might

£76
l Advanced Use of consider bidding 5♦, but since you can
the Basics be pretty sure that your partner’s suits West North East South
are spades and clubs you have a better 2♣ Pass
l Weak Two Openings option available. You have a decent 2♦ Pass 3♠ Pass
l Strong Two Openings fit with spades and plenty of ruffing ?
l Defence to Weak Twos potential, so you should bid 4♠.
4♥. Over the 2♣ opening, you do have
l Defence Against 1NT
an 8-card suit, but you take the sensible
l Doubles 11. Dealer West. Game All. option of making a negative response
l Two Suited Overcalls ♠ A K Q J 10 4 3 2 ♠ Void first – after all, you might find you have
♥ 4 3 N ♥ A 9 2 only one useful card for your partner.
l Defending against Other W E
♦ A K Q ♦ 9 8 7 6 5 East’s jump to 3♠ is exactly what was
Systems S
♣ Void ♣ 10 8 7 5 2 discussed in question 11. East is telling
l Misfits and Big you that his spades are so good that he is
Distributions setting them as the trump suit, even if you
Operating system requirements: West North East South have a void. He does not want to hear
Windows only 2♣ Pass 2♦ Pass about your long suits at all, just your high
? cards.
Therefore, over 3♠ you bid 4♥
Mr Bridge Limited 3♠. You have an extraordinary hand – showing your ace. Partner bids
( 01483 489961 and you need to get this across to your Blackwood and knows that your one ace
www.mrbridge.co.uk/shop partner – you clearly have game in your is in hearts, rather than in clubs, so can
own hand, so you open 2♣ to show that bid 6♠ with confidence. n

BRIDGE October 2020 Page 51

You might also like